RD hepatobiliary formatted Flashcards

1
Q
  1. 10cm pancreatic cyst (cystic mass) in a young patient (25 years). (Incidental with no calcifications). Most likely is
    a. Serous
    b. Mucinous
    c. Solid pseudopapillary
    d. Insulinoma
    e. Gastrinoma
A

c. Solid pseudopapillary T Young age suggests solid pseudopapillary neoplasm with predominate cystic degeneration. RG 2011 – F»M (9:1), young adult (mean age 25); typically large (mean 9cm); 30% have calcifications. Against this is no mention of solid components or haemorrhage

.1. 10cm pancreatic cyst (cystic mass) in a young patient (25 years). (Incidental with no calcifications). Most likely is (Lim DDx p208) (SK)

a. Serous F mean age 65 years; multiple small cysts (cut orange); maybe at this age if they had VHL?
b. Mucinous F mean age 40-60 years; smaller number of larger (> 2cm) cysts
c. Solid pseudopapillary T Young age suggests solid pseudopapillary neoplasm with predominate cystic degeneration. RG 2011 – F»M (9:1), young adult (mean age 25); typically large (mean 9cm); 30% have calcifications. Against this is no mention of solid components or haemorrhage.
d. Insulinoma F 4th-6th decades; usually small & hypervascular solid
e. Gastrinoma F 4th-5th decades; somewhat larger but hypervascular solid

How well did you know this?
1
Not at all
2
3
4
5
Perfectly
2
Q
  1. A small diverticulum arises from the common bile duct, separate from the ampulla. Which is the correct classification?
    a. I
    b. II
    c. III
    d. IV
    e. V
A

b. II

How well did you know this?
1
Not at all
2
3
4
5
Perfectly
3
Q
  1. Woman on HRT is jaundiced, with abnormal LFTs and non-dilated ducts on ultrasound. (LAS – liver normal echogenicity). Most likely cause:
    a. Steatohepatitis
    b. Cholestasis
    c. Hepatocellular necrosis
A

B: Cholestasis.
*LW: Estrogen therapy can induce a clinically cholestatic liver injury during first few cycles, insideous onset with jaundice, elevated liver enzymes, with bland intra hepatic cholestasis and biopsy showing little inflammation or necrosis. Thus favoured answer is B - cholestasis given Hx stating HRT, jaundice, abnormal LFTs and normal liver echogenicity.

b. Cholestasis ?T = systemic retention of bilirubin, bile salts & cholesterol due to inadequate biliary elimination of these solutes. Results from hepatocellular dysfunction or biliary obstruction. In this patient intrahepatic cholestasis could account for the findings.
3. Woman on HRT is jaundiced, with abnormal LFTs and non-dilated ducts on ultrasound. (LAS – liver normal echogenicity). Most likely cause: (SK)

a. Steatohepatitis ?T NASH = hepatocytes filled with fat vacuoles with inflammatory infiltrates -> probably wrong -> liver would be echogenic
b. Cholestasis ?T = systemic retention of bilirubin, bile salts & cholesterol due to inadequate biliary elimination of these solutes. Results from hepatocellular dysfunction or biliary obstruction. In this patient intrahepatic cholestasis could account for the findings.
c. Hepatocellular necrosis ?T in the setting of hepatitis?Robbins says “estrogen replacement therapy” can cause a cholestatic liver injury.No other reference anywhere for effects of HRT on the liver – apart from a number saying it can reduce risks of type 2 DM and improve LFTs.Robbins p860: Individuals with simple steatosis are generally asymptomatic. AST & ALT elevated in 90% of patients with NASH.

How well did you know this?
1
Not at all
2
3
4
5
Perfectly
4
Q
  1. Middle aged woman with pancreatic head mass. Least likely to be a?
    a. Solid pseudopapillary neoplasm
    b. Serous cystadenoma
    c. Mucinous cystic neoplasm
    d. Islet cell tumour
    e. Pseudocyst
A

a. Solid & pseudopapillary neoplasm F age < 35 years (rare in adults)

  1. Middle aged woman with pancreatic head mass. Least likely to be a?
    a. Solid & pseudopapillary neoplasm F age < 35 years (rare in adults)
    b. Serous cystadenoma mean age 65 years; multiple small cysts; favours the pancreatic head
    c. Mucinous cystic neoplasm mean age 40-60 years; smaller number of larger (> 2cm) cysts; favours the pancreatic body/tail
    d. Islet cell tumour 4th-6th decades; usually small & hypervascular solid
    e. Pseudocyst
How well did you know this?
1
Not at all
2
3
4
5
Perfectly
5
Q
  1. Young woman. CT triple phase. Arterial and PV phase, heterogenous enhancement. Us heterogenous. Most likely
    a. adenoma
    b. FNH
    c. HCC
A

A – adenoma = T young females on OCP; heterogenous rapid enhancement (hyperdense arterially, but often isodense on pre-con, PV & delayed phases); heterogenous/variable at US

A – adenoma = T young females on OCP; heterogenous rapid enhancement (hyperdense arterially, but often isodense on pre-con, PV & delayed phases); heterogenous/variable at US

B – FNH = F young females, homogenous arterial enhancement (except central scar); US homogenously hypoechoic except for central scar

C – HCC = F – early enhancement +/- central necrosis; typically echogenic at US; can be radiologically identical to adenoma, but less common in young patients without cirrhosis

How well did you know this?
1
Not at all
2
3
4
5
Perfectly
6
Q
  1. Patient with diarrhoea. Hypokalaemia. Pan shows arterial and PV base enhancing mass. Most likely.
    a. Gastrinoma
    b. VIPoma
    c. Somastotinoma
A
B = VIPoma = WHDA syndrome – watery diarrhoea, hypokalaemia, achlorhydria
A = gastrinoma = 2nd most common islet cell neoplasm; causes ZES (diarrhoea, PUD, but no hypokalaemia)
B = VIPoma = WHDA syndrome – watery diarrhoea, hypokalaemia, achlorhydria
C = Somatostatinoma = Diabetes, diarrhoea, cholelithiasis
How well did you know this?
1
Not at all
2
3
4
5
Perfectly
7
Q
  1. Definition of segment 4a.
A

above portal vein, between middle and left hepatic vein

How well did you know this?
1
Not at all
2
3
4
5
Perfectly
8
Q
patient with cholangiocarcinoma extends beyond right and left hepatic ducts
a. type 1 
b 2
c. 3a
d. 3 b
e. 4
A

e. 4

Bismuth-Corlette classification — Cancers arising in the perihilar region have been further classified according to their patterns of involvement of the hepatic ducts (the Bismuth-Corlette classification):

  • Tumor of CHD below the confluence of the left and right hepatic ducts (Type I)
  • Tumor of CHD reaching the confluence (Type II)
  • Tumors occluding the common hepatic duct and either the right or left hepatic duct (Types IIIa and IIIb, respectively)
  • Tumors that are multicentric, or that involve the confluence and both the right and left hepatic ducts (Type IV)

Classified based on anatomy and radiography
- Peripheral (10%) Intrahepatic; proximal to secondary biliary radicles
- Perihilar (50%) Klatskin tumor: Hilar tumor involving the confluence of hepatic ducts
- Distal (40%) Extrahepatic; distal CBD
May arise as short stricture or small polypoid mass

How well did you know this?
1
Not at all
2
3
4
5
Perfectly
9
Q
  1. A 45 year old man has a 3 cm homogenous pancreatic mass. 3 lesions are present in the liver. A lesion in segment IV measures 2cm, is well circumscribed and an-echoic with posterior acoustic enhancement. A lesion in segment VI measures 3cm and is heterogenous in appearance. A lesion in segment VIII measures 2cm, is well circumscribed and an-echoic with posterior acoustic enhancement. The referring oncologist needs tissue for diagnosis. Which of the following is most appropriate:
    a. Imaging guided biopsy of the segment IV lesion.
    b. Imaging guided biopsy of the segment VI lesion.
    c. Imaging guided biopsy of the segment VIII lesion.
    d. Imaging guided biopsy of the pancreas.
    e. Progress CT in 3 months
A

b. Imaging guided biopsy of the segment VI lesion. T may be a pancreatic NET with mets?, alternatively pancreatic adenocarcinoma

*LW:
Biopsying a liver lesion easier to do than transcolemic FNA of the pancreatic mass.
Furthermore, risk, although accuracy is still undetemrined, in peritoneal seeding with FNA biopsy of pancreas, and accurracy is also relatively reduced.
It is an accepatable method to prove non surgical candidate in pancreatic Ca.
ERCP brushings or endoluminal US biopsy are methods with higher accuracy.
–> thus why segment VI lesion image guided biopsy is answer, and will likely confirm metastatic pancreatic ca.

*AJL - Biopsying the pancreatic mass will tell you its a cancer (which we kind of already know) but will not tell you if there is metastatic disease.
Biopsying the bad looking liver lesion will tell you if there is metastatic disease or not which is the real clinical question. (overall I agree with the above but just providing an explanation)

How well did you know this?
1
Not at all
2
3
4
5
Perfectly
10
Q
  1. Which of the following CT sequences will be most informative if looking for a hepatoma:
    a. CT with biliary contrast
    b. CT with IV contrast in the arterial phase
    c. CT with IV contrast in the portal venous phase
    d. CT with IV contrast at a 10 minute delay
    e. CT with no IV contrast
A

b. CT with IV contrast in the arterial phase T best for detection – early enhancement during arterial phase.
- -> HCC

How well did you know this?
1
Not at all
2
3
4
5
Perfectly
11
Q
  1. A 45 year-old woman presents with 3 incidental liver lesions which are hyperechoic on ultrasound and measure between 3-5cm. They MOST LIKELY represent:
    a. Mulitfocal HCC
    b. Focal nodular hyperplasia
    c. Adenoma
    d. Metastasis
    e. Haemangioma.
A

d. Metastasis T most common liver mass in adults; echogenic if hypervascular mets, calcified mets
e. Haemangioma.T 80% in females (more common postmenopause); 2nd most common liver mass after metastases; echogenic; usually solitary

Without any history of malignancy, go E haemangioma?

  1. A 45 year-old woman presents with 3 incidental liver lesion which are hyperechoic on ultrasound and measure between 3-5cm. They MOST LIKELY represent:
    a. Mulitfocal HCC T but unlikely without Hx of cirrhosis
    b. Focal nodular hyperplasia F usually homogenously hypoechoic
    c. Adenoma F variable US, classically hyperechoic with central anechoic areas
    d. Metastasis T most common liver mass in adults; echogenic if hypervascular mets, calcified mets
    e. Haemangioma.T 80% in females (more common postmenopause); 2nd most common liver mass after metastases; echogenic; usually solitary
How well did you know this?
1
Not at all
2
3
4
5
Perfectly
12
Q
  1. A middle age male presents for staging CT scan of cholangiocarcninoma, which involves the primary confluence but not the secondary confluence. Which grade?
    a. I
    b. II
    c. IIIA
    d. IIIB
    e. IV
A

b. II

Bismuth-Corlette classification — Cancers arising in the perihilar region have been further classified according to their patterns of involvement of the hepatic ducts (the Bismuth-Corlette classification):

  • Tumors below the confluence of the left and right hepatic ducts (Type I)
  • Tumors reaching the confluence (Type II)
  • Tumors occluding the common hepatic duct and either the right or left hepatic duct (Types IIIa and IIIb, respectively)
  • Tumors that are multicentric, or that involve the confluence and both the right or left hepatic duct (Type IV)
How well did you know this?
1
Not at all
2
3
4
5
Perfectly
13
Q
  1. Pancreatic mass in 50 year old female, 9cm, mural nodule on MRI, Ca++ in wall on CT, most correct
    a. Mucinous tumour
    b. Macrocystic serous tumour
    c. Microcystic serous tumour
    d. Pseudo solidpapillary tumour
    e. Intraductal papillary tumour
A

a. Mucinous tumour T cyst + mural nodule. Peripheral eggshell calcification is specific for mucinous cystic neoplasm & is highly predictive of malignancy (RG 05). Mean age 50 years. Size 2-12cm. Surgical removal indicated

.5. Pancreatic mass in 50 year old female, 9cm, mural nodule on MRI, Ca++ in wall on CT, most correct

a. Mucinous tumour T cyst + mural nodule. Peripheral eggshell calcification is specific for mucinous cystic neoplasm & is highly predictive of malignancy (RG 05). Mean age 50 years. Size 2-12cm. Surgical removal indicated.
b. Macrocystic serous tumour F no solid component
c. Microcystic serous tumour F no solid component, septal/central calcification
d. Pseudo solid papillary tumour F cyst + mural nodule; 5-30% have peripheral calcification; often large; younger age group (< 35 yrs)
e. Intraductal papillary tumour F calcification unlikely; can have mural nodule

How well did you know this?
1
Not at all
2
3
4
5
Perfectly
14
Q
  1. Female with rising LFTs, RUQ pain has a hyperechoic liver parenchyma and normal biliary tree, most likely explanation?
    a. Cholestasis
    b. Hepatocellular necrosis
    c. Cirrhosis
    d. Fibrosis
A

b. Hepatocellular necrosis T?? talking about hepatitis – can feature hepatocellular necrosis.

How well did you know this?
1
Not at all
2
3
4
5
Perfectly
15
Q
  1. steroid using guy with jaundice. Normal biliary tree
    a. steatohepatitis
    b. cholestasis
    c. cirrhosis.
    d. fatty liver
A

IF THEY MEAN ANABOLIC STEROIDS… ANSWER IS B (or D?)
Pure (canalicular, bland or noninflammatory) cholestasis is characterized by cholestasis with very little hepatocellular inflammation. Bile plugging is frequently seen, predominantly in zone 3 hepatocytes or canaliculi. Aminotransferases are generally normal or minimally elevated, with greater elevations in alkaline phosphatase and γ-glutamyl transferase. This type of injury is often seen with the use of anabolic steroids or oral contraceptives. Drugs causing this type of injury interfere with hepatocyte secretion of bile constituents and other pigment and dye substances via the bile salt excretory protein (BSEP). Chronic cholestasis may evolve from acute drug-induced cholestasis.Emedicine:Anabolic steroid abuse has been considered a risk factor for nonalcoholic fatty liver disease.

IF THEY MEAN GLUCOCORTICOIDS… ANSWER IS D Drug-induced chronic steatosis is predominantly macrovesicular in contrast to the microvesicular steatosis usually seen in acute drug injury. Serum aminotransferases are typically moderately elevated. Drugs associated with these types of injury include chemotherapeutic agents (eg, 5-fluorouracil, cisplatin, tamoxifen), chloroform, diethylstilbestrol, ethanol, glucocorticoids, gold, griseofulvin, methotrexate, mercury, nifedipine, nitrofurantoin, NSAIDs (eg, ibuprofen, indomethacin, sulindac), tamoxifen, and total parenteral nutrition [44,77,78]. These medications rarely cause steatohepatitis but may exacerbate or precipitate it in patients with risk factors for NASH.

How well did you know this?
1
Not at all
2
3
4
5
Perfectly
16
Q
  1. Guy in MVA. splenic and diaphragmatic rupture. Nodules in the left lung that takes up Tc99 sulphur colloid. most likely
A

splenosis
Mettler – Nuc Med p203
- 99mTc sulfur colloid is the most commonly used agent in N/M for imaging the liver & spleen
- Uptake by reticuloendothelial system (phagocytes)
o Liver (80-90%, Kupffer cells)
– uptake & distribution reflect distribution of functioning reticuloendothelial cells & distribution of hepatic perfusion

o	Spleen (5-10%, phagocytes)
o	Bone marrow (small amount)
-	Normal = homogenous distribution throughout liver
-	Liver lesions differentiation:
o	Increased uptake
•	FNH
•	Cirrhosis with regenerative nodule
•	Budd-chiari (caudate lobe)
•	SVC obstruction (quadrate lobe)
o	Decreased uptake
•	Metastasis (esp. colon)
•	Cyst
•	HCC, esp. in cirrhosis
•	Adenoma
•	Haematoma
•	Haemangioma
•	Abscess
•	Pseudotumour (cirrhosis)
•	Cholangiocarcinoma 
•	Liver abscess
•	Focal fatty infiltration

o Sites of abnormal (non liver/spleen) uptake
• Renal transplant (rejection – current or previous)
• Diffuse lung activity – multiple causes (Table 8-3 p207)

How well did you know this?
1
Not at all
2
3
4
5
Perfectly
17
Q
  1. Hyperechoic lesion on us in liver. takes up labelled rbc. Most likely
A

cavernous haemangioma

Mettler p211
• 99mTc labelled RBC scan
• Haemangioma highly likely if:
o Increased activity after labelled red cell scan (often need significant delay to show)
o Decreased uptake on sulphur colloid scan

How well did you know this?
1
Not at all
2
3
4
5
Perfectly
18
Q
  1. guy with 3cm pancreatic head mass. segment 4b cystic lesion. segment 6 hyperechoic heterogenous lesion. segment 8 hyper echoic lesion. which one do you biopsy
A

probably hyper echoic lesion??

How well did you know this?
1
Not at all
2
3
4
5
Perfectly
19
Q

35 yo woman. liver MRI. moderate arterial enhancement. Washout portal venous phase. Central high T2. Most likely

a. FNH
b. adenoma.
c. fibrolamellar HCC

A

ANS = A = FNH (central high T2 = scar)[central scar in fibrolamellar HCC usually hypointense on all MR sequences, rarely T2 hyper]Note large haemangiomas can have a central scar also.

How well did you know this?
1
Not at all
2
3
4
5
Perfectly
20
Q

1) Liver Segment III: (note previous exam segment II)
i) Above portal vein and to left of left hepatic vein
ii) Below portal vein and to left of left hepatic vein
iii) Above portal vein and between middle and left hepatic vein
iv) Above portal vein and to right of to right hepatic vein
v) Below portal vein and between middle and left hepatic veins

A

ii) Below portal vein and to left of left hepatic vein

How well did you know this?
1
Not at all
2
3
4
5
Perfectly
21
Q

2) Liver Segment IVa:
i) Above portal vein and to left of left hepatic vein
ii) Below portal vein and to left of left hepatic vein
iii) Above portal vein and between middle and left hepatic veins
iv) Above portal vein and to right of right hepatic vein
v) Below portal vein and between middle and left hepatic veins

A

iii) Above portal vein and between middle and left hepatic veins

How well did you know this?
1
Not at all
2
3
4
5
Perfectly
22
Q

3) Thick-walled, septated collection in liver post diverticular abscess drainage by surgeons. Best option:
i) IV abs only
ii) Needle aspiration
iii) Percutaneous insertion 5fr drain
iv) Percutaneous insertion 10fr drain
v) Surgical drainage

A

iv) Percutaneous insertion 10fr drain TKandarpa p529- Commonly use 7-14 Fr catheters- Larger catheters used for bigger, more complex collections (e.g. 12-14 Fr – theorectically allow more free drainage)

How well did you know this?
1
Not at all
2
3
4
5
Perfectly
23
Q

4) Chronic pancreatitis
i) Calcification is acinar
ii) 15% go on to pancreatic Ca
iii) 60% of pseudocysts in acute pancreatitis resolve within 6 weeks
iv) Peritoneal calcification is a recognised feature

A

iv) Peritoneal calcification is a recognised feature T?
4) Chronic pancreatitis
i) Calcification is acinar F The ducts become obstructed by proteinaceous plugs that can eventually accumulate calcium carbonate. This obstruction results in ductal ectasia and periductal fibrosis. The calculi occur in ducts of all sizes and vary from microscopic to greater than 1 cm in diameter. (AJR 02). Calcification occurs in 40-60% of patients with alcoholic pancreatitis (StatDx), and 75% of cases in USA are due to alcoholism.
ii) 15% go on to pancreatic Ca F 2-4% (Dahnert)
iii) 60% of pseudocysts in acute pancreatitis resolve within 6 weeks F Pseudocyst occurs in 10-20% of cases of acute pancreatitis, usually ≥ 4 weeks. Spontaneous resolution in 25-40% (StatDx), no time frame given (presumably would be longer). AJR 2011 states 40% resolve spontaneously.
iv) Peritoneal calcification is a recognised feature T?

How well did you know this?
1
Not at all
2
3
4
5
Perfectly
24
Q

Liver Segment II

a. Above portal vein and medial to medial hepatic vein
b. Below portal vein and medial to medial hepatic vein
c. Above portal vein and between middle and medial hepatic vein
d. Above portal vein and lateral to lateral hepatic vein

A

A

How well did you know this?
1
Not at all
2
3
4
5
Perfectly
25
Q

Hepatic adenoma. Does it have uptake on sulfar colloid scan???

A

Not much. FNH takes up more sulfar colloid.

o	Increased uptake
•	FNH
•	Cirrhosis with regenerative nodule
•	Budd-chiari (caudate lobe)
•	SVC obstruction (quadrate lobe)
o	Decreased uptake
•	Metastasis (esp. colon)
•	Cyst
•	HCC, esp. in cirrhosis
•	Adenoma
•	Haematoma
•	Haemangioma
•	Abscess
•	Pseudotumour (cirrhosis)
•	Cholangiocarcinoma 
•	Liver abscess
•	Focal fatty infiltration

o Sites of abnormal (non liver/spleen) uptake
• Renal transplant (rejection – current or previous)
• Diffuse lung activity – multiple causes (Table 8-3 p207)

How well did you know this?
1
Not at all
2
3
4
5
Perfectly
26
Q

Young man. abnormal LFT. investigation of choice.

a. ERCP
b. MRCP
c. AXR
d. US

A

Ultrasound

How well did you know this?
1
Not at all
2
3
4
5
Perfectly
27
Q
  1. NOT a differential Dx for hyperintense lesion of non-con T1 MRI of the liver?
    a. Hethaemaglobin
    b. Cavernous hemangioma
    c. Hepatoma
    d. Metastasis
    e. Angiomyolipoma
A

b. Cavernous hemangioma F? iso-hypointense on T1, bright on T2 (light bulb) – however if contains haemorrhage, with contain T1 hyperintensity (although uncommon & not listed in StatDx)
1. NOT a differential Dx for hyperintense lesion of non-con T1 MRI of the liver?

a. Hethaemaglobin T
b. Cavernous hemangioma F? iso-hypointense on T1, bright on T2 (light bulb) – however if contains haemorrhage, with contain T1 hyperintensity (although uncommon & not listed in StatDx)
c. Hepatoma T
d. Metastasis T haemorrhage, melanoma, mucinous (colon, ovary, pancreas), myeloma
e. Angiomyolipoma T macroscopic fat is T1 hyperintense

How well did you know this?
1
Not at all
2
3
4
5
Perfectly
28
Q
  1. Which is NOT a recognised cause of Budd-Chiari syndrome
    a. Chronic pancreatitis
    b. Bone marrow transplantation
    c. Chemotherapy
    d. Systemic lupus erythematosus
    e. Oral contraceptives
A

a. Chronic pancreatitis
• Primary: Venous outflow membranous obstructiono Controversial etiology
o Congenital, injury, or infection

• Secondary: Thrombotic, rarely nonthrombotic
o Obstruction of central and sublobular veins
o Obstruction of major hepatic veins
o Obstruction of small centrilobular veins (venoocclusive disease)
o Nonthrombotic: Hepatic & extrahepatic masses

Causes: idiopathic (2/3 (other websites suggest on 1/3 idiopathic).
Thrombosis - hypercoagulable state (5P’s: PCV, pill, pregnancy, increased plts, PNH), or injury to vessel wall (XRT, chemo/IS in BMT patients, Jamaican bush tea) Non-thrombotic - tumour extension into IVC (RCC, HCC), IVC diaphragm (congenital, acquired), right atrial tumour, constrictive pericarditis, RHF. [Dahnert; SLE RG 2004]

How well did you know this?
1
Not at all
2
3
4
5
Perfectly
29
Q
  1. Which of the following proposed conditions is LEAST LIKELY to be associated with primary biliary cirrhosis
    a. Hepatic osteodystrophy
    b. Choledochocele
    c. Lymphadenopathy
    d. HCC
    e. Gallstones
A

b. Choledochocele – F
1. Which of the following proposed conditions is LEAST LIKELY to be associated with primary biliary cirrhosis

a. Hepatic osteodystrophy T - a generic definition for the metabolic bone disease that may occur in individuals with chronic liver disease. Osteopaenia occurs in PBC.
b. Choledochocele – F
c. Lymphadenopathy – T more prominent feature in PBC than in other aetiologies of CLD (StatDx); nodes seen within porta & portocaval regions
d. HCC increased T – a complication, but less common than with viral or alcoholic cirrhosis & tend to be well-differentiated (StatDx)
e. Gallstones T 30-40%Primary biliary cirrhosis- idiopathic condition- progressive sclerosis of the biliary tree- non - suppurative

How well did you know this?
1
Not at all
2
3
4
5
Perfectly
30
Q
  1. Which of the following statements regarding gas lucencies in the liver on plain radiograph of abdomen is MOST CORRECT?
    a. Gas may be seen in the liver following chemoembolisation of a liver tumor
    b. PV gas is a common finding in hemorrhagic pancreatitis
    c. Biliary tree gas is a recognised finding after infarction of bowel
    d. PV gas is predominantly centrally located
A

a. Gas may be seen in the liver following chemoembolisation of a liver tumor T

  1. Which of the following statements regarding gas lucencies in the liver on plain radiograph of abdomen is MOST CORRECT?
    a. Gas may be seen in the liver following chemoembolisation of a liver tumor T
    b. PV gas is a common finding in hemorrhagic pancreatitis F but can occur as a complication of acute pancreatitis (StatDx – “less common” cause)
    c. Biliary tree gas is a recognised finding after infarction of bowel F
    d. PV gas is predominantly centrally located F peripheral, mobile
How well did you know this?
1
Not at all
2
3
4
5
Perfectly
31
Q
  1. Liver lesion with a central stellate area that is hypointense on T1 and T2 imaging. What is the most likely cause?
  2. Hepatic adenoma
  3. FNH
  4. HCC
  5. Fibrolamaller HCC
  6. Hemangioma
A

4.Fibrolamaller HCC - T - central scar hypointense on T1 & T2. (=FLC)

  1. Liver lesion with a central stellate area that is hypointense on T1 and T2 imaging. What is the most likely cause? (JS/SK)
  2. Hepatic adenoma - F - heterogenous appearance but no central scar
  3. FNH – F (but ?T as ‘most likely’) – SK: central scar T1 hypo & T2 hyperintense (StatDx, Lin DDx p183)JS: central scar is hyperintense on T2, but 25% can be hypointense. Size of the lesion would also be important as the main DDx is FLHCC.
  4. HCC - F - variable appearance
  5. Fibrolamaller HCC - T - central scar hypointense on T1 & T2. (=FLC)
  6. Hemangioma - F - T2 hyperintense with nodular enhancement from periphery. Large (giant) haemangiomas can have central scar – T1 hypointense & T2 hyperintense

Pearls from StatDx on FLC:
• FLC: Bigger, more heterogeneous mass, frequently with calcified central/eccentric scar and features of malignancy (vessel/biliary obstruction, nodal invasion, and lung metastases)
• Scar on T2WI: Hypointense (FLC), hyperintense (FNH)
• FLC = Large, heterogeneous, hypervascular tumor in young adult

How well did you know this?
1
Not at all
2
3
4
5
Perfectly
32
Q
  1. Clinical scenario of diarrhoea, flushing etc. Portal venous CT suggests mesenteric lesion with desmoplastic reaction and liver mets. What is the next most appropriate investigation?
  2. MRI
  3. MIBG scan
  4. Octreotide scan
  5. Triple phase CT abdomen
  6. USS pelvis
A
  1. Octreotide scan - T – see flow chart below. As CT ‘suggests’ lesion with mets, the diagnosis is still not confirmed. The next examination would probably actually be serum metanephrines.
  2. Clinical scenario of diarrhoea, flushing etc. Portal venous CT suggests mesenteric lesion with desmoplastic reaction and liver mets. What is the next most appropriate investigation? (JS, GC, TW)
  3. MRI - F
  4. MIBG scan - F - uptake in 44-63%.
  5. Octreotide scan - T – see flow chart below. As CT ‘suggests’ lesion with mets, the diagnosis is still not confirmed. The next examination would probably actually be serum metanephrines.
  6. Triple phase CT abdomen - F - liver lesions already thought to be mets 5.USS pelvis - F
How well did you know this?
1
Not at all
2
3
4
5
Perfectly
33
Q
  1. Alcoholic male patient has 4cm hypoechoic mass on ultrasound in segment 4 background of echoic liver. It is decided that a biopsy is needed to further manage pt. INR 3.0 platelet count 40,000. What is the best method to do biopsy.
  2. Open biopsy
  3. Laproscopic biopsy
  4. Percutaneous Core biopsy
  5. Percutaneous FNA
  6. Transjugular biopsy
A
  1. Percutaneous Core biopsy - T – treat with FFP first.
  2. Alcoholic male patient has 4cm hypoechoic mass on ultrasound in segment 4 background of echoic liver. It is decided that a biopsy is needed to further manage pt. INR 3.0 platelet count 40,000. What is the best method to do biopsy? (GC, TW). Probably incomplete recall
  3. Open biopsy - F
  4. Laproscopic biopsy - F - likely to have a higher diagnostic yield in patients with cirrhosis compared to percutaneous liver bx and is excellent for staging the extent of disease in patients with various intra-abdominal malignancies. However, it requires general anesthesia, which is associated with increased risk and cost.
  5. Percutaneous Core biopsy - T – treat with FFP first.
  6. Percutaneous FNA - F – (potentially true) useful when a specific lesion needs to be sampled but produces only small numbers of cells.
  7. Transjugular biopsy - F – this would be true if investigating diffuse liver disease (rather than a mass) as the main advantage of the transjugular approach to liver biopsy is that the liver tissue is obtained within the vascular system, thereby minimizing the risk of bleeding. The transjugular approach is most commonly used for patients in whom a percutaneous approach is considered to be unsafe or in whom a hepatic vascular procedure is required for other reasons (such as for a venogram). [UpToDate].
34
Q
  1. Which is not a normal embryological variant?
  2. PV aneurysm
  3. Shunt b/n PV and IVC
  4. PV duplication
  5. Portosystemic shunt
A
  1. Fistulation between PV and IVC F - this is the aim of TIPS for patients with portal HTN (diversion of blood from portal circulation, through hepatic parenchyma, to the systemic circulation; usually between right hepatic vein and right PV).
  2. Which is not a normal embryological variant? (GC)
  3. PV aneurysm T - incomplete regression of the distal right vitelline vein leading to a diverticulum that would ultimately develop into an aneurysm in the proximal SMV could explain aneurysms in that location. An inherent weakness of the vessel wall is another theory proposed to support a congenital origin. The most common locations are the portal confluence, main PV, and intrahepatic PV branches at bifurcation sites.
  4. Fistulation between PV and IVC F - this is the aim of TIPS for patients with portal HTN (diversion of blood from portal circulation, through hepatic parenchyma, to the systemic circulation; usually between right hepatic vein and right PV).
  5. SMA/SMV reversal in position in gut malrotation T - SMV to the left of SMA.
  6. Intrahepatic portosystemic shunt T - persistence of an omphalomesenteric venous system with the right horn of the sinus venosus and rupture of a congenital aneurysm of the portal vein into a hepatic vein are congenital conditions that have been proposed to explain intrahepatic portosystemic shunts.
  7. PV duplication T - Congenital anomalies of the main PV include prepancreatic portal vein, which is frequently associated with situs inversus and other congenital malformations; double PV; congenital agenesis of the PV; and congenital agenesis of the major branches of the portal vein. [Congenital & Acquired Anomalies of the PV system, RG 2002]
35
Q
  1. Haemangiomas have a number of atypical features. Which is most unlikely?
  2. 1cm subcapsular lesion which rapidly fills and is isodense on PV phase
  3. Low signal septa on T2 imaging
  4. Peripheral puddling, non-continuous enhancement on arterial imaging
  5. 3cm lesion with capsular retraction
  6. Homogeneous arterial enhancement
A

4.3cm lesion with capsular retraction F – (GC)

  1. Haemangiomas have a number of atypical features. Which is most unlikely? (GC/SK)
  2. 1cm subcapsular lesion which rapidly fills and is isodense on PV phase T . Common in subcapsular area in posterior right lobe of liver
  3. Low signal septa on T2 imaging T – hypointense internal septa in giant haemangioma (StatDx).
  4. Peripheral puddling, non-continuous enhancement on arterial imaging T - classic.
  5. 3cm lesion with capsular retraction F – (GC)SK - ? T – listed in Lin DDx p199 as DDx for hepatic capsular retraction – can occur in haemangiomas > 4cm
  6. Homogeneous arterial enhancement T - seems to occur significantly more in small lesions, i.e. “flash haemangioma” (42% of those <1cm). Atypical patterns include: echoic border on USlarge heterogeneous (giant if >4cm)rapidly filling – occurs in 16%, more often in small lesions (42% of hemangioma <1cm)calcifiedrarely: hyalinized (thought to represent the end-stage of involution), cystic/multilocular, fluid-fluid level, pedunculated. [Atypical Hemangiomas of Liver, RG 2000]
36
Q
  1. In which case would you notify the team?
  2. Portal venous gas post UVC placement
  3. UVC passing caudally at the umbilical level
  4. UVC tip at porta hepatis
  5. UAC tip at T10
  6. A calcified hepatic mass developing since removal of UVC
A
  1. UVC passing caudally at the umbilical level - T – this would indicate the UVC is positioned in the umbilical artery.
  2. In which case would you notify the team? (GC)
  3. Portal venous gas post UVC placement - F - air embolism at the time of catheterisation is not uncommonly seen on the initial radiograph (due to poor technique). The air is an isolated finding and will be absorbed. Conversely, PV air associated with NEC will typically be accompanied by pneumatosis intestinalis.
  4. UVC passing caudally at the umbilical level - T – this would indicate the UVC is positioned in the umbilical artery.
  5. UVC tip at porta hepatis – F (less true) - the preferred location of the tip of the UVC is in the cephalad portion of the IVC or at the IVC-RA junction. Leaving the catheter in the portal vein or injecting hyperosmolar solutions into the PV contributes to the development of hepatic necrosis or PV thrombosis. The tip of the UVC can coil in the umbilical recess before entering the left portal vein. If after coiling in the umbilical recess, the UVC is advanced farther, it may travel in a retrograde direction in the umbilical vein toward the umbilicus. Rarely, the UVC may reach the IVC but may progress caudally rather than cranially in the IVC.
  6. UAC tip at T10 - F - a high location is acceptable. A lower location (tip between the L3/4 interspace and the superior portion of hte body of L5, below the major aortic branches) is preferred because less catheter surface is available to accumulate thrombin clot. Placement with the tip in the proximal common iliac a. is also acceptable. Classically “dips” into the pelvis before passing cranially. Incorrect placement (ie. into a branch of the aorta) may lead to thrombosis and organ necrosis.
  7. A calcified hepatic mass developing since removal of UVC - F - If the UVC perforates an intrahepatic vascular wall, a hepatic hematoma may result; these may eventually calcify. Thrombus may also calcify.
37
Q
  1. The following statements are true in relation to hepatic adenomas?
  2. Uptake is seen on sulphur colloid scans in the majority.
  3. Uptake of SPIO is a recognized feature.
  4. 50% have fat on CT.
  5. 60% have calcification on CT.
  6. Majority associated with OCP use.
A

5.Majority associated with OCP use. T - 2.5x risk after 5-year use, 25x risk >9-year use. Other risk factors include type I glycogen storage disease and use of anabolic steroids.

  1. The following statements are true in relation to hepatic adenomas? (GC/SK)
  2. Uptake is seen on sulphur colloid scans in the majority. F - Kupffer cells are often found in adenomas but in reduced numbers and with little or no function. Focal photopenic lesion seen on Tc-99m sulphur colloid scans; 23% may show uptake equal to or slightly less than liver. Also, usually no increased activity on HIDA scan (cf. FNH); and no gallium uptake. Key sulphur colloid positive lesion is FNH.
  3. Uptake of SPIO is a recognized feature. F - usually don’t show uptake of superparamagnetic iron oxide particles, resulting in decreased SI on T2WI. Also, usually no substantial uptake of Gd-BOPTA (Multihance), lesions appear hypointense on delayed imaging.
  4. 50% have fat on CT. F - mass of decreased density due to fat and areas of necrosis in < 20% (statdx SK).
  5. 60% have calcification on CT. F - uncommonly calcified (< 20%). Calcifications usually seen within cystic portions that represent areas of necrosis or old haemorrhage.
  6. Majority associated with OCP use. T - 2.5x risk after 5-year use, 25x risk >9-year use. Other risk factors include type I glycogen storage disease and use of anabolic steroids.
38
Q
  1. In liver MRI which of the following shows loss of signal in out of phase imaging?
  2. Adenoma.
  3. FNH.
  4. HCC.
  5. Mets.
  6. Pseudotumour.
A
  1. Adenoma. T - adenoma cells are larger than normal hepatocytes and contain large amounts of glycogen and lipid. Intra- and extracellular lipid uncommonly manifests as macroscopic fat deposits within the tumour (7%). In contrast, 35-77% of adenomas demonstrate steatosis at chemical shift imaging.
  2. FNH F (Lin – rare) - presence of fat is extremely rare; usually patchy in distribution; intratumoural steatosis may or may not be assocd with diffuse hepatic steatosis.
  3. HCC. T (but less fat than in adenomas) - larger tumours have patchy fatty metamorphosis, seen as macroscopic fat on CT. Fatty change can be seen in up to 35% of small HCCs, seen as a signal drop on chemical shift imaging.
  4. Mets. F (Lin – rare) - in general, liver mets do not contain fat. Exceptional examples include VIPoma.
  5. Pseudotumour. F - majority occur in Asians, children/young adults, right lobe; may contain calcification, larger lesions may have central necrosis. Variable MR appearance, usually T1 hypointense / T2 hyperintense cf. skeletal muscle, and heterogeneously enhance. Cause is unknown, some believe it is a low-grade fibrosarcoma with inflammatory (lymphomatous) cells; usually contains both T and B-cells (cf. clonal population in lymphoma). HCC - 35% in small HCC (less fat then adenoma)
39
Q

10.LIVER Associations, which is most correct?

  1. Peliosis hepatis is associated with HIV and Aids.
  2. HCC is associated with OCP.
  3. Selective hypertrophy of segments 3 and 4 occur in Budd –Chiari.
  4. Adenomatous hyperplastic nodules typically enhance during arterial phase following IV contrast.
A
  1. Peliosis hepatis is associated with HIV and AIDS. T - peliosis hepatis is a rare benign disorder characterised by multiple blood-filled cysts. Steroids, OCP, tamoxifen, oestrogens, Ig therapy, and azthioprine are usually assocd with PH. It has also been reported in pts with haem/onc disorders (Hodgkin’s, MM), transplantation, chronic infection (esp. pulmonary TB and HIV). Rochilimaea henselae or Bartonella henselae are bacterial agents causative for peliosis in immunodeficient patients that can be treated with antibiotics. [Pseudotumoural appearance of PH, AJR 2005]
  2. HCC is associated with OCP less correct - several studies have demonstrated that prolonged use (esp. >8yrs) and higher synthetic oestrogen content increases the relative risk of adenoma and HCC. The effect of adenoma formation may be reversible with discontinuation, but HCC still can occur after resolution of the adenoma. The absolute risk of HCC is small, but the RR of 2.6 in case-control studies could have implications in societies where OCP use is prevalent and other RFs are uncommon, esp. because the risk of HCC does not decrease for at least 10yrs after prolonged OCP use. [HCC diagnosis & treatment, BI Carr 2005]
  3. Selective hypertrophy of segments 3 & 4 occur in Budd-Chiari. F - caudate hypertrophy.
  4. Adenomatous hyperplastic nodules typically enhance during arterial phase following IV contrast. F - ie. a low-grade dysplastic nodule (high-grade being an atypical adenomatous hyperplasia). Regenerative nodules: low T2, variable T1, no enhancement on arterial phase imaging. HCC: high T2, variable T1, intense arterial enhancement. The SI and enhancement characteristics of dysplastic nodules are not yet well established; but I assume they would not “typically” enhance (unless they are large-? or contain a focus of HCC). [Benign vs Malig Hepatic Nodules RG 2002]
40
Q

11.Regarding Caroli’s disease (which is false)?

  1. There is an association with renal medullary cystic disease.
  2. There is an increased risk of HCC.
  3. Associated with ARPCKD.
  4. Tc99 HIDA scan is typically normal
  5. There is marked predisposition to biliary calculus disease
A
  1. There is a markedly increased risk of HCC. F *LW: i cannot find a quoted increased risk of HCC other than isolated case reports, while regularly documented cholangiocarcinoma risk, as per Robbins (7%).
  2. Tc-99m HIDA scan is typically normal F Unusual pattern of retained activity throughout liver (StatDx)
    * AJL agrees with the answers above.
  3. Regarding Caroli’s disease true/false? (GC) [SK – which is false]
  4. There is an association with renal medullary cystic disease. F assoc/ w/ MSK &; ARPKD [SK – medullary cystic disease includes MSK & sporadic nephronphthisis, so I would give this true, as MSK is assoc/ w/ Caroli]
    * LW: confusing terms: Robbins states: two major types of medullary cystic disease: medullary sponge kidney and nephronophthisis-medullary cystic disease complex. Most Caroli’s resources (StatDx, Radiopedia) quote associations with Medullary Sponge Kidney, and Autosomal recessive and dominant polycystic kidney disease. Thus if only one option was to be correctly false, the other listed options are false, hence this would not be the chosen answer. However, if other options were correct, then I would thus favour this answer to be false, as I presume the question would then be wanting known association of the above specifc diseases, rather than broad pathology term (but this is least likely….)
  5. There is a markedly increased risk of HCC. F [SK – true – 100 x’s increased risk] - increased risk for cholangiocarcinoma (7% ). *LW: i cannot find a quoted increased risk of HCC other than isolated case reports, while regularly documented cholangiocarcinoma risk, as per Robbins.
  6. Associated with ARPKD. T - patients with ARPKD also have congenital hepatic fibrosis. This is due to ductal plate malformation (double-layered sleeve of hepatocyte precursor cells that forms around the portal veins, remodels abnormally resulting in increased no. of bile ducts with abn branching and variable dilatation). Part of DPM also consists of fibrosis of the portal tracts. It is hypothesized that CHF and Caroli’s represent a spectrum of portal tract malformations (CHF = small duct involvement with portal fibrosis; Caroli’s = large duct ectasia without fibrosis).
  7. Tc-99m HIDA scan is typically normal F Unusual pattern of retained activity throughout liver (StatDx)
  8. There is a marked predisposition to biliary calculous disease T - Cx include bile stasis with recurrent cholangitis, predominantly bilirubin calculi, and liver abscess. [ARPKD, RG 2000; Dahnert]
41
Q

12.The following are true regarding MRCP?

  1. Highly accurate in the diagnosis of pancreatic divisum.
  2. Has an accuracy of about 75% in the diagnosis of CBD calculi in most series.
  3. Is accurate in the diagnosis of early pancreatitis.
  4. Is best performed using a gradient echo T1 weighted sequence.
  5. Is unlikely to be successful if the serum bilirubin is 5 times normal.
A

1.Highly accurate in the diagnosis of pancreatic divisum. T - has been shown to be highly sensitive and specific for pancreas divisum. Non-communication of the dorsal and central ducts, independent drainage sites and a dominant dorsal pancreatic duct can be seen using MRCP. Depiction can be improved using dynamic MRCP after secretin stimulation. [Singapore Med J 2008]

  1. The following are true regarding MRCP? (GC)
  2. Highly accurate in the diagnosis of pancreatic divisum. T - has been shown to be highly sensitive and specific for pancreas divisum. Non-communication of the dorsal and central ducts, independent drainage sites and a dominant dorsal pancreatic duct can be seen using MRCP. Depiction can be improved using dynamic MRCP after secretin stimulation. [Singapore Med J 2008]
  3. Has an accuracy of about 75% in the diagnosis of CBD calculi in most series. F - MRCP is well suited to the diagnosis of choledocholithiasis because CBD stones appear as low-signal intensity foci within the high-SI bile. Stones as small as 2 mm can be detected. Sensitivity 81-100%; specificity 85-100%; PPV 82-100%; NPV 94-100%. In view of the high NPV of MRCP, the primary utility in the setting of choledocholithiasis may lie not in the detection of CBD stones but in their exclusion.
  4. Is accurate in the diagnosis of early pancreatitis. F - In the severe forms of the disease, imaging is performed to assess the perfusion of the pancreatic parenchyma, the extent of necrosis, and the presence and extent of fluid collections. CECT is currently considered the standard reference for evaluating these parameters; it also presents a more manageable environment for severely ill patients. However, contrast-enhanced MRI/MRCP can also accomplish these goals and have recently been suggested as an alternative to CECT for the initial staging of acute pancreatitis.
  5. Is best performed using a gradient echo T1 weighted sequence. F - Fast GRE T1-weighted sequence is used as the 3-plane localiser. Imaging the pancreatic duct is dependent on heavily T2WI that selectively displays static or slow-moving fluid-filled structures. Two different and complementary approaches are generally used for MRCP: a thick-slab, single-shot TSE T2-weighted sequence and a multisection thin-slab, single-shot TSE T2-weighted sequence.
  6. Is unlikely to be successful if the serum bilirubin is 5 times normal. F - potentially an issue with CT cholangiography.[MRCP, RG 1999; MRI pancreas, RG 2002]
42
Q
  1. CT findings in Cancer of the head of pancreas, which is most correct:
  2. Enhancement of the lesion during the arterial phase after IV contrast
  3. There is typically preservation of parenchyma in body and tail of pancreas.
  4. Accuracy of CT with regard to predicting that a tumor is resectable approaches 100%.
  5. Accuracy with regard to predicting that a tumor is non-resectable is about 90-100% in most recent series.
  6. Obstruction of both the CBD and main pancreatic duct is specific for malignancy in the pancreatic head.
A
  1. Accuracy with regard to predicting that a tumor is non-resectable is about 90-100% in most recent series. T
  2. CT findings in Cancer of the head of pancreas, which is most correct: (GC) CME02.47
  3. Enhancement of the lesion during the arterial phase after IV contrast F - hypovascular lesion, with maximal attenuation difference between tumour & pancreas best seen during parenchymal (late arterial) phase. PV phase is ideal for detection of liver mets. About 10% of pancreatic adenoCa are isoattenuating relative to the background parenchyma.
  4. There is typically preservation of parenchyma in body and tail of pancreas. F - atrophy of body and tail is an indirect sign (20%).
  5. Accuracy of CT with regard to predicting that a tumor is resectable approaches 100%. F - pancreatic adenoCa infiltrates lymphatic vessels early, and local infiltrative disease can manifest as subtle infiltration of peripancreatic tissue. This local invasion can cause underestimation of the true extent and stage of the tumor and is a cause of aborted surgical resection if not identified preoperatively.
  6. Accuracy with regard to predicting that a tumor is non-resectable is about 90-100% in most recent series. T
  7. Obstruction of both the CBD and main pancreatic duct is specific for malignancy in the pancreatic head. F - “double duct sign” is a reliable indicator of an obstructing lesion, although it is not specific for pancreatic adenocarcinoma.TNM staging:T1 <2cm; T2 >2cm - both confined to pancreasT3 extends beyond pancreas but doesn’t involve vesselsT4 involves either coeliac axis or SMAN1 involves regional nodesM1 distant mets Stage I (T1 or T2) and stage II (T1-2N1, T3N0-1) are resectable.Stage III (T4) and stage IV (M1) are unresectable.[Preop Assessment of Pancreatic Ca, RG 2007]
43
Q
  1. US Paediatric liver

1. Echogenic mass with cysts and calcification is compatible with hepatoblastoma

A

1.Echogenic mass with cysts and calcification is compatible with hepatoblastoma Large, well-defined and heterogeneous liver mass in an infant. At US heterogeneous with cysts & calcification. Most common in infants, peak 1-2 years, range neonate – 15 years.

44
Q
  1. Which is false about SLE

1. pancreatitis is due to secondary vasculitis

A

*LW:
Robbins states the ischaemic arm of the pancreatitis pathogenesis pathways includes SLE which causes a vasculitis. So I would take this as true.

Further details (can ignore):
Acute pancreatitis occurs in 2 to 8 percent of patients with systemic lupus erythematosus.
In the majority of cases, the development of acute pancreatitis occurs in the setting of active SLE.
The presentation of pancreatitis does not differ from patients without SLE, and includes acute onset of severe persistent epigastric pain often radiating to the back and a three times or greater elevation in serum amylase and lipase.
The pathogenetic mechanism of pancreatitis related to SLE is unknown, but vascular damage from vasculitis or thrombosis (often in association with antiphospholipid antibodies) has been proposed.
Risk factors to the development of pancreatitis, as in patients without SLE, include gallstones, alcohol use, infection, and hypertriglyceridemia

  1. Which is false about SLE: (GC)
  2. pancreatitis is due to secondary vasculitis ?? Pancreatitis may be due to vasculitis, ischaemia of small pancreatic vessels, immune complex deposition, or a combination of these entities. In the case of haemorrhagic pancreatitis, a necrotizing vasculitis due to obliterative vascular thrombosis has been implicated.
45
Q
  1. Regarding pancreatic neuroendocrine tumours, which is true:
  2. Insulinomas are usually <15mm at diagnosis
  3. Somataostatinomas are associated with NHDA syndrome
  4. Insulinomas don’t enhance
  5. Insulinomas are frequently ectopic
  6. Glucagonoma is more common than insulinoma
A

1.Insulinomas are usually <15mm at diagnosis T

  1. Regarding pancreatic neuroendocrine tumours, which is true: (GC)
  2. Insulinomas are usually <15mm at diagnosis T
  3. Somataostatinomas are associated with NHDA syndrome F - VIPoma are assocd with the WDHA syndrome = watery diarrhoea, hypoK+, achlorhydia (VIP inhibits gastrin production); more recently and more accurately described as WDHH (…hypochlorhydia or “pancreatic cholera”).
  4. Insulinomas don’t enhance F - 66% are hypervascular at angiography; variable at CT (hypo/iso/hyper); hyperintense on T2 and dynamic C+ images, tumours >2cm may show ring enhancement.
  5. Insulinomas are frequently ectopic F - 2-5% in ectopic location. 10% are multiple (esp in MEN 1).
  6. Glucagonoma is more common than insulinoma F - uncommon tumour derived from alpha cells; insulinoma is the most common syndromic islet cell tumour.
46
Q
  1. Spleen, which is false:
  2. Haemangiomas are the most common benign lesion
  3. On non contrast CT the spleen is isodense to liver
  4. In-phase MRI shows decreased intensity
  5. Haemangioma is bright on T2
  6. Hamartomas enhance post contrast
A
  1. On non contrast CT the spleen is isodense to liver F - homogeneous, typically 40-60HU on NECT; attenuation is normally 5-10HU less than that of liver.
  2. Spleen, which is false: (GC)
  3. Haemangiomas are the most common benign lesion T - most common primary splenic tumour.
  4. On non contrast CT the spleen is isodense to liver F - homogeneous, typically 40-60HU on NECT; attenuation is normally 5-10HU less than that of liver.
  5. In-phase MRI shows decreased intensity T - recalled question read “on opposed (or was it in phase?) MRI decreased intensity”.
  6. Haemangioma is bright on T2 T - hypo/iso on T1, hyper on T2; hypointense areas due to haemosiderin deposits; progressive centripetal enhancement with persistent uniform enhancement on delayed images.
  7. Hamartomas enhance post contrast T - benign asymptomatic lesions, usually single; commonly associated with tuberous sclerosis. Most are heterogeneously hyperintense on T2; with diffuse enhancement on early postcontrast images and more uniform enhancement on delayed images.
47
Q

21.Regarding TIP, which is true:

  1. Ascites is a contraindication
  2. Increased velocity by 50cm/sec compared with baseline on a followup study indicates possible stenosis
  3. Retrograde flow in the intrahepatic portal veins indicates stenosis
  4. Need to have same pressure as SVC for best result
  5. Left common femoral approach best
A

2.Increased velocity by 50cm/sec compared with baseline on a followup study indicates possible stenosis - T –

  1. Regarding TIP, which is true: Transjugular intrahepatic portosystemic shunt (TW)
  2. Ascites is a contraindication - F - if it was, there probably wouldn’t be many TIPS procedures done. See below for contraindications.
  3. Increased velocity by 50cm/sec compared with baseline on a followup study indicates possible stenosis - T – change in 50cm/sec from baseline is concerning for stenosis. Elevated maximum and depressed minimum stent velocities are signs of stent stenosis (one system uses 90cm/sec and 190cm/sec as the lower and upper limits, of normal stent velocities). Additional signs of dysfunction are low PV velocity, a temporal increase or decrease in max and min stent velocities on sequential examinations, and reversal of flow in the draining hepatic vein (US requisites).
  4. Retrograde flow in the intrahepatic portal veins indicates stenosis - F – If this was talking about reversal of baseline flow in the PV after TIPS this would also be correct. Retrograde flow is normal post TIPS: when first placing the TIPS, the stent decompresses the portal system directly into the low pressure hepatic venous system, portal flow in the right and left portal vein usually reverses after stent placement and is directed into the stent instead of into the liver.
  5. Need to have same pressure as SVC for best result - F
  6. Left common femoral approach best - F - transjugular. Wire through heart into MHV. Then ream through the liver to where you expect the PV to be, and if you miss - try again. Some use a transcutaneous needle into PV and leave a little marker or wire to provide a target for the ‘reaming’, some use US.

TIPS is an expensive, invasive procedure with significant morbidity and mortality. It requires close follow-up due to hight incidence of shunt stenosis, and recurrent portal HTN. TIPS does not affect the outcome of liver transplantation. Indications:Active hemoirrhage despite emergent endoscopic treatmentRecurrent variceal hemorrhage despite adequate endoscopic treatmentBleeding isolated varices in gastric fundus (potential indication)Refractory ascites (potential indication)Contraindications: Not to be used for primary prophylaxis of variceal hemorrhage. Prehepatic portal hypertension (eg PV thrombosis, tumor at porta hepatis) as obstruction is up-stream. Not to be used to simply increase platelet count in cirrhotics with thrombocytopenia (cirrhotics get splenic engorgement and platelet sequenstration, and can see improvement in thrombocytopenia post TIPS).Patients with pulmonary hypertension and hepatopulmonary syndrome (Post TIPS get immediate increase in venous return to heart. Outcome of TIPS in these patients is not predictable, and may be detrimental).Portal hypertension associated with polycystic liver disease or Caroli’s disease (creation of intrahepatic tract may involve traversal of a cyst, which is associated with a high risk of severe hemorrhage).

48
Q
  1. Budd Chiari syndrome, which is true:
  2. Spider like appearance of portal veins on portal venography is diagnostic
  3. 70% is idiopathic
  4. TIPS is not an option for treatment
  5. Central increased uptake is seen on sulphur colloid scan
A

4.Central increased uptake is seen on sulphur colloid scan - T – (SCS was used previously, but has generally been replaced by other imaging modalities). Hypertrophy of the caudate lobe associated with BCS is demonstrated by its preferential uptake of the radionuclide. However, more than half of patients with BCS do not demonstrate this pattern (UTD). Sulfur colloid liver scan may show a central area of increased activity believed to be secondary to enlargement of the caudate lobe (AJR). See below. (note that this could be considered false as some don’t think the central uptake is actually increase, but normal with relative reduced flow around).

  1. Budd Chiari syndrome, which is true: (TW) – controversial
  2. Spider like appearance of portal veins on portal venography is diagnostic - F – “Spider web” pattern of hepatic venous collaterals.
  3. 70% is idiopathic - F - in approx 75% of pts, a hematological abnormality or a cause of thrombotic diathesis can be identified that predisposes the patient to the occurrence of BCS (Radiographics 2009). Underlying disorcer can be identified in over 80% of patients with BCS (UTD). (note that Dahnert says 66% idiopathic) - so this could have been true in years past.
  4. TIPS is not an option for treatment - F - TIPS is a good alternative method for relieving liver congestion, with lower mortality and morbidity than those assoc with open surgical procedures. Other options: medical, thrombolysis, angioplasty + stent of stenoses.
  5. Central increased uptake is seen on sulphur colloid scan - T – (SCS was used previously, but has generally been replaced by other imaging modalities). Hypertrophy of the caudate lobe associated with BCS is demonstrated by its preferential uptake of the radionuclide. However, more than half of patients with BCS do not demonstrate this pattern (UTD). Sulfur colloid liver scan may show a central area of increased activity believed to be secondary to enlargement of the caudate lobe (AJR). See below. (note that this could be considered false as some don’t think the central uptake is actually increase, but normal with relative reduced flow around).
49
Q
  1. Caroli’s disease, which is false:
  2. Increased risk of biliary tree malignancy
  3. Associated with choledochocele
  4. Associated with increased risk of ascending cholangitis
  5. Increased risk of abscess
  6. Extrahepatic duct is normal
A
  1. Associated with choledochocele - F - Choledochoceles refer to a specific type of choledochal cyst (type III under the Todani classification system). Choledochocele can be congenital or acquired (ie stone passage + stenosis).
  2. Caroli’s disease, which is false: (note Caroli’s disease vs Caroli’s syndrome) (TW)
  3. Increased risk of biliary tree malignancy - T - increased risk of cholangiocarcinoma (up to 7%), probably due to the significant bile stasis and the presence of high concentrations of uncongjugated secondary bile salts.
  4. Associated with choledochocele - F - based on Tadani classification Type V choledochal cyst represents Caroli disease (but see below). Choledochocele = cystic dilatation of the distal / intramural duodenal portion of the CBD with herniation of the CBD into the duodenum (similar to ureterocele). Choledochocele can be congenital or acquired (ie stone passage + stenosis).
  5. Associated with increased risk of ascending cholangitis - T - predisposes to stagnation of bile leading to formation of biliary sludge and intraductal lithiasis. Bacterial cholangitis occurss frequently and may be complicated by septicemia and hepatic abscess formation.
  6. Increased risk of abscess - T - see ans 3.
  7. Extrahepatic duct is normal - T - in both Caroli’s disease and Caroli’s syndrome, the biliary abnormality consists of segmental, saccular dilatations of the large intrahepatic bile ducts. May be limited to one lobe, most commonly left lobe. Althought the Todani classification of choledochal cysts includes Caroli’s disease as a type V choledoochal cyst, the pathogenesis of Caroli’s disease (AR and often assoc with renal disorders) makes these disorders unrelated.CD is a congenital disorder characterized by multifocal, segmental dilatation of large intrahepatic bile ducts. The condition is usually associated with renal cystic disease of varying severity. Caroli initially described 2 variants, which has lead to some confusion in terminology:Caroli’s disease = less common form. Characterised by bile ductular ectasia without other apparent hepatic abnormalities.Caroli’s syndrome = more common variant. Bile duct dilatation is associated with congenital hepatic fibrosis. Most cases (CD) are transmitted by an autosomal recessive fashion and are assoc with ARPKD. There have been rare cases occurring with ADPKD. Caroli = 6 letters = Chromosome 6.
50
Q
  1. Pancreas, which is false?
  2. Macroadenoma always malignant
  3. Microadenoma usually benign
  4. Most cysts are pseudocysts
  5. Enhancing wall of pseudocysts does not necessarily mean infection
A
  1. Macroadenoma always malignant - F - No malignant potential for serous types (see ans 2). Macrocystic variant of serous cystadenoma is difficult to distinguish from mucinous tumor which is a low-grade malignant tumor
  2. Pancreas, which is false? (TW)
  3. Macroadenoma always malignant - F - No malignant potential for serous types (see ans 2). Macrocystic variant of serous cystadenoma is difficult to distinguish from mucinous tumor which is a low-grade malignant tumor.
  4. Microadenoma usually benign - T - glycogen-rich or micro-/macrocystic serous adenoma. Arises from centroacinar cell. Honeycombed or spongy appearance on cut section. Serous cystadenomas: no malignant potential (Blackbook).
  5. Most cysts are pseudocysts - T - pseudocyst = fluid collection >4wks old and surrounded by a defined wall. Pseudocysts are the most common cystic lesions of the pancreas. All other cystic lesions including cystic neoplasms represent only 10-15% of pancreatic cysts.
  6. Enhancing wall of pseudocysts does not necessarily mean infection - T - pseudocysts usually have a smooth thin wall or a thick wall with uniform thickness. Normally there is enhancement of the thin rim of fibrous capsule and no enhancement of pseudocyst contents. Infection, haemorrhage, inflammation all would also cause enhancement.
51
Q
  1. Which is true re: Budd Chiari syndrome, best answer:
  2. 70% aetiology unknown
  3. Hypertrophy of caudate in the majority
  4. Spiderweb appearance on hepatic artery on angio
  5. Tc99m Sulphur colloid shows predominantly central uptake in liver
  6. TIPS is a recognized treatment
A

5.TIPS is a recognized treatment - T - TIPS is a good alternative method for relieving liver congestion, with lower mortality and morbidity than those assoc with open surgical procedures.

  1. Which is true re: Budd Chiari syndrome, best answer: (TW)
  2. 70% aetiology unknown - F - in approx 75% of pts, a hematological abnormality or a cause of thrombotic diathesis can be identified that predisposes the patient to the occurrence of BCS (Radiographics 2009). Underlying disorder can be identified in over 80% of patients with BCS (UTD).(note that Dahnert says 66% idiopathic)
  3. Hypertrophy of caudate in the majority - F - tend to see hypertrophy in chronic phase of BCS, not acute phase. The frequency of caudate lobe hypertrophy in BCS has not been accurately established, however some authors report that CL is markedly enlarged in about 50% of pts with this condition. Only see typical increased uptake by enlarged CL with scintigraphy in about 17% or pts (AJR 2003).
  4. Spiderweb appearance on hepatic artery on angio - F - “Spider web” pattern of hepatic venous collaterals.
  5. Tc99m Sulphur colloid shows predominantly central uptake in liver - F – not best answer. Hypertrophy of the caudate lobe associated with BCS is demonstrated by its preferential uptake of the radionuclide. However, more than half of patients with BCS do not demonstrate this pattern (UTD).
  6. TIPS is a recognized treatment - T - TIPS is a good alternative method for relieving liver congestion, with lower mortality and morbidity than those assoc with open surgical procedures.
52
Q
  1. Which is true re: Cystic liver lesions:
  2. Hydatid cysts demonstrate daughter cysts in 70%
  3. Rim calcification in hydatid cysts implies death of parasite
  4. Patients with ADPK disease invariably develop liver cysts (Invariably = always / in every case)
  5. Hepatic cysts are assoc with tuberous sclerosis
  6. Echinococcus multilocularis is the usual organism in hydatid cysts
A

1.Hydatid cysts demonstrate daughter cysts in 70% - T

  1. Which is true re: Cystic liver lesions: (TW)
  2. Hydatid cysts demonstrate daughter cysts in 70% - T
  3. Rim calcification in hydatid cysts implies death of parasite - F - dense calcification may be assumed to indicate the death of the cyst. Partially calcified cysts are still alive. Death of the parasite is not necessarily indicated by calcification of the pericyst, it is implied by complete calcification.
  4. Patients with ADPK disease invariably develop liver cysts - F – hepatic cysts develop later than renal cysts in ADPKD. Prevalence increases with age 30yo (10-20%), >60yo (50-70%).
  5. Hepatic cysts are assoc with tuberous sclerosis F – TS get adenomas and lipomyomas in liver
  6. Echinococcus multilocularis is the usual organism in hydatid cysts - F – there are 2 types of echinococcus (tapeworm) infections. E.granulosis is the more common type, whereas E. multilocularis is less common but more invasive.
53
Q
  1. Which is false re: TIPS
  2. It is a recognized treatment of choice for refractory ascites
  3. Contraindicated in the presence of RHF
  4. May cause biliary dilation acutely
  5. IJV thrombosis precludes performance
A

4.IJV thrombosis precludes performance - F - RIJV usually used, LIJV can be used if right side can’t be used. Femoral venous approach techniques have also been described, but are used less commonly, and are technically more demanding (eMed).

  1. Which is false re: TIPS (TW)
  2. It is a recognized treatment of choice for refractory ascites - T
  3. Contraindicated in the presence of RHF - T - right-sided heart failure with increased central venous pressure (eMed). Pulmonary HTN and hepatopulmonary syndrome is a contraindication.
  4. May cause biliary dilation acutely - T - potential complication of fistula to biliary tree. Studies have also shown biliary epithelial proliferation with creation of large cystlike spaces (but this is not acute).
  5. IJV thrombosis precludes performance - F - RIJV usually used, LIJV can be used if right side can’t be used. Femoral venous approach techniques have also been described, but are used less commonly, and are technically more demanding (eMed).
54
Q
  1. Which of the following statements regarding detection of colorectal liver metastases, which is true:
  2. Haemangiomas are increasingly bright on T2
  3. CTAP via a SMA injection is associated with fewer non tumorous perfusion defects than via a splenic artery injection
  4. Delayed hepatic CT, 10-14 days after intrahepatic arterial administration of iodinised poppyseed oil (Lipiodol) is a useful technique
  5. Intraoperative ultrasonography has the highest spatial resolution of all currently applied imaging techniques
  6. Transient peripheral rim enhancement is typical of metastases on IV gadolinium enhanced T1 weighted MR
A

1.Haemangiomas are increasingly bright on T2 - T - “light bulb” appearance on T2, markedly hyperintense with intensity increasing with echo time due to slow flowing blood. Also see “light bulb” sign with cystic and neuroendocrine mets.

55
Q
  1. Of the pancreas, which is false:
  2. Increased size of pancreatic duct with increasing age
  3. Reflux into the pancreas equals obstruction at the ampulla
  4. CBD always empties through the ampulla of Vater
  5. Main duct joins CBD in 70%
A

2.Reflux into the pancreas equals obstruction at the ampulla - F - anomalous pancreatiobiliary junction = fusion of pancreatic duct and CBD outside the duodenal wall. Presence of a long common channel may allow reflex of pancreatic secretions into biliary system and vice versa.

  1. Of the pancreas, which is false: (TW)
  2. Increased size of pancreatic duct with increasing age - T – Primer: (N) 3mm young adult, 5mm elderly. Normal size (Radiographics) 3.5mm head, 2.5mm body, 1.5mm tail.
  3. Reflux into the pancreas equals obstruction at the ampulla - F - anomalous pancreatiobiliary junction = fusion of pancreatic duct and CBD outside the duodenal wall. Presence of a long common channel may allow reflex of pancreatic secretions into biliary system and vice versa.
  4. CBD always empties through the ampulla of Vater - T - think it is T, embryologically part of the hepatic bud.
  5. Main duct joins CBD in 70% - T – note that this was previously given as false (with 2. having “??” as explanation). Although number a bit low, pancreas divisum (MRCP below) occurs in about 7% of population where CBD and main pancreatic duct are separate. Probably some other variants too. Primer says 95% join.
56
Q
  1. Asymptomatic 30 y.o. female US shows a 3cms slightly hypoechoic and homogenous mass, no cirrhosis. CT shows hypodense lesion on non con, isodense on pv phase, most likely diagnosis:
  2. HCC
  3. FNH
  4. Haemangioma
  5. Fibrolamellar HCC
  6. Adenoma
A
  1. FNH - T - hamartomatous malformation (contains hepatocytes, Kupffer cells, bile ducts without connection to biliary tree); well-circumscribed non-encapsulated nodular mass in a non-cirrhotic liver. 85% are <5cm. Multiple in 20% of cases. US: iso- / mildly hypo- / mildly hyperechoic, homogeneous. +/- hyperechoic central scar.NECT: iso- / slightly hypoattenuating and homogenous. CECT: transient enhancement in arterial phase, isodense on PV and delayed phases. If a central scar is present (50%), it will be hypodense on arterial, and enhance in delayed phase . MRI: T1 iso- / hypo, T2 slightly hyper- / isointense; central scar T1 hypo, T2 hyper or hypo (dpt on vascular channels and oedema); Primovist uptake on delayed phase (functional hepatocytes).
  2. Asymptomatic 30 y.o. female US shows a 3cms slightly hypoechoic and homogenous mass, no cirrhosis. CT shows hypodense lesion on non con, isodense on pv phase, most likely diagnosis: (GC & TW)
  3. HCC - F
  4. FNH - T - hamartomatous malformation (contains hepatocytes, Kupffer cells, bile ducts without connection to biliary tree); well-circumscribed non-encapsulated nodular mass in a non-cirrhotic liver. 85% are <5cm. Multiple in 20% of cases. US: iso- / mildly hypo- / mildly hyperechoic, homogeneous. +/- hyperechoic central scar.NECT: iso- / slightly hypoattenuating and homogenous. CECT: transient enhancement in arterial phase, isodense on PV and delayed phases. If a central scar is present (50%), it will be hypodense on arterial, and enhance in delayed phase . MRI: T1 iso- / hypo, T2 slightly hyper- / isointense; central scar T1 hypo, T2 hyper or hypo (dpt on vascular channels and oedema); Primovist uptake on delayed phase (functional hepatocytes).
  5. Haemangioma - F - echogenic in 80%
  6. Fibrolamellar HCC - F
  7. Adenoma - F - rare and symptomatic in >80%
57
Q
  1. Lesion with the same imaging characteristic as above, with known breast primary:
  2. Metastasis
  3. FNH
  4. Adenoma
  5. Haemangioma
  6. None of the above
A

33.Lesion with the same imaging characteristic as above, with known breast primary: (TW, GC, JS)

  1. Metastasis - ?T – breast can be hypervascular, and mets would need to be considered and excluded.
  2. FNH - ?T - anecdotally - if breast mets were isodense on PV phase - we must be missing a hell of a lot of staging / f/u breast cancer scans. Agreed, however, that mets would have to be excluded with known breast primary - raises the qu of how liver lesion was detected in the 1st place and for what reason (ie why doing a liver US, or CT KUB, or UECT phase). ** open for debate **
  3. Adenoma
  4. Haemangioma
  5. None of the above
58
Q

34.Which is true regarding pancreatic tumours:

  1. In adenocarcinoma, enhancement of the lesion during arterial phase is typical
  2. In pancreatic head carcinoma, typically there is preservation of parenchyma in the body and the tail
  3. Microcystic adenomas have a strong malignant potential
  4. Accuracy with regard to predicting that a pancreatic carcinoma is non resectable is >90% in most recent series
  5. In pancreatic carcinoma, involvement of 25% of the circumference of the portal vein on CT indicates a 75% likelihood that the tumour is non respectable
A
  1. Accuracy with regard to predicting that a pancreatic carcinoma is non respectable is >90% in most recent series - T - CT good for determining non-resectability, average for resectability. PPV of helical CT for unresectability range between 89% to 100%, however PPV for resectability are 45-79%.
  2. Which is true regarding pancreatic tumours: (TW)
  3. In adenocarcinoma, enhancement of the lesion during arterial phase is typical - F - AKA pandreatic ductal carcinoma / pancreatic cancer. Heterogeneous, poorly enhancing mass. Occurs in head 60%, body 20%, diffuse 15%, tail 5%.
  4. In pancreatic head carcinoma, typically there is preservation of parenchyma in the body and the tail - F - Parenchymal atrophy distal to tumor may be seen.
  5. Microcystic adenomas have a strong malignant potential - F - serous macro- / microcysadenomas. Benign pancreatic tumor, no malignant potential (Blackbook).
  6. Accuracy with regard to predicting that a pancreatic carcinoma is non respectable is >90% in most recent series - T - CT good for determining non-resectability, average for resectability. PPV of helical CT for unresectability range between 89% to 100%, however PPV for resectability are 45-79%.
  7. In pancreatic carcinoma, involvement of 25% of the circumference of the portal vein on CT indicates a 75% likelihood that the tumour is non resectable - F - 50% tends to be the cut off from what I can see. For those with more than one-half circumferential involvement of one or more of the majroy vessels or occlusion of the superior mesenteric vein or the SMV-PV confluence, we consider the lesion categorically unresectable (UTD).
59
Q
  1. In Caroli’s disease, which is false:
  2. There is an association with renal medullary cystic disease
  3. There is an association with hepatic fibrosis
  4. There is marked predisposition to biliary calculus disease
  5. There is a markedly increased risk of HCC
A

4.There is a markedly increased risk of HCC - F - increased risk for cholangiocarcinoma (7%)

  1. In Caroli’s disease, which is false: (GC & TW)
  2. There is an association with renal medullary cystic disease - T - in 80%
  3. There is an association with hepatic fibrosis - T - could argue this one. Caroli’s disease vs Caroli’s syndrome. Caroli initially described 2 variants, which has lead to some confusion in terminology: Caroli’s disease = less common form. Characterised by bile ductular ectasia without other apparent hepatic abnormalities. Caroli’s syndrome = more common variant. Bile duct dilatation is associated with congenital hepatic fibrosis (UTD).
  4. There is marked predisposition to biliary calculus disease - T - predisposes to stagnation of bile leading to formation of biliary sludge and intraductal lithiasis. Bacterial cholangitis occurss frequently and may be complicated by septicemia and hepatic abscess formation
  5. There is a markedly increased risk of HCC - F - increased risk for cholangiocarcinoma (7%)
60
Q
  1. The following statements regarding CT findings in cancer of the pancreatic head are true:
  2. Enhancement of the lesion during the arterial phase after IV contrast is typical
  3. There is typically preservation of the parenchyma in the body and the tail of the pancreas
  4. Accuracy of CT with regard to predicting that a tumour is resectable approaches 100%
  5. Accuracy with regard to predicting that a tumour is non resectable is about 90-100% in most recent series
A
  1. Accuracy with regard to predicting that a tumour is non resectable is about 90-100% in most recent series - T - see ans 3.
  2. The following statements regarding CT findings in cancer of the pancreatic head are true: (TW)
  3. Enhancement of the lesion during the arterial phase after IV contrast is typical - F - AKA pandreatic ductal carcinoma / pancreatic cancer. Heterogeneous, poorly enhancing mass. Occurs in head 60%, body 20%, diffuse 15%, tail 5%. 20% distal atrophy.
  4. There is typically preservation of the parenchyma in the body and the tail of the pancreas - F - Parenchymal atrophy distal to tumor may be seen.
  5. Microcystic adenomas have a strong malignant potential - F - serous macro- / microcysadenomas. Benign pancreatic tumor, no malignant potential (Blackbook).
  6. Accuracy of CT with regard to predicting that a tumour is resectable approaches 100% - F - CT good for determining non-resectability, average for resectability. PPV of helical CT for unresectability range between 89% to 100%, however PPV for resectability are 45-79%.
  7. Accuracy with regard to predicting that a tumour is non resectable is about 90-100% in most recent series - T - see ans 3.
61
Q
  1. The following statements related to endocrine tumours of the pancreas are true:
  2. Gastrinomas are malignant in about 10% of cases
  3. About 60% of insulinomas are benign
  4. About 40% of insulinomas are extrapancreatic in location
  5. Most tumours show no enhancement on dynamiuc contrast enhanced CT
  6. About 70% of insulinomas are less than 15mm in diameter
A

5.About 70% of insulinomas are less than 15mm in diameter - T - avg tumor size 1-2cm; <1.5cm in 70% (Dahnert).

  1. The following statements related to endocrine tumours of the pancreas are true: (TW)
  2. Gastrinomas are malignant in about 10% of cases - F – multiple and malignant 60% (Blackbook)
  3. About 60% of insulinomas are benign - F – 90%, 10% malignant
  4. About 40% of insulinomas are extrapancreatic in location - F - no predilection for any part of pancreas, 2-5% in ectopic location; 10% mutliple (Dahnert).
  5. Most tumours show no enhancement on dynamiuc contrast enhanced CT - F - syndromic islet cell tumors are generally <3cm in size, typically hyperenhancing and are usually best seen in arterial phase. Non-syndromic ICTs tend to be larger.
  6. About 70% of insulinomas are less than 15mm in diameter - T - avg tumor size 1-2cm; <1.5cm in 70% (Dahnert).
62
Q
  1. Liver and biliary imaging:
  2. Pancreatic islet cell metastases are best evaluated with PV phase CT
  3. Peliosis hepatis is associated with AIDS
  4. The majority of patients with untreated Wilson disease demonstrate increased attenuation of the liver on CT
A

2.Peliosis hepatis is associated with AIDS - T - peliosis hepatis is a rare benign disorder characterised by multiple blood-filled cysts. Steroids, OCP, tamoxifen, oestrogens, Ig therapy, and azthioprine are usually assocd with PH. It has also been reported in pts with haem/onc disorders (Hodgkin’s, MM), transplantation, chronic infection (esp. pulmonary TB and HIV). Rochilimaea henselae or Bartonella henselae are bacterial agents causative for peliosis in immunodeficient patients that can be treated with antibiotics. [Pseudotumoural appearance of PH, AJR 2005]

  1. Liver and biliary imaging: (TW & GC)
  2. Pancreatic islet cell metastases are best evaluated with PV phase CT - F - islet cell tumor & mets are hypervascular and are hyperenhancing and are usually best seen in arterial phase.
  3. Peliosis hepatis is associated with AIDS - T - peliosis hepatis is a rare benign disorder characterised by multiple blood-filled cysts. Steroids, OCP, tamoxifen, oestrogens, Ig therapy, and azthioprine are usually assocd with PH. It has also been reported in pts with haem/onc disorders (Hodgkin’s, MM), transplantation, chronic infection (esp. pulmonary TB and HIV). Rochilimaea henselae or Bartonella henselae are bacterial agents causative for peliosis in immunodeficient patients that can be treated with antibiotics. [Pseudotumoural appearance of PH, AJR 2005]
  4. The majority of patients with untreated Wilson disease demonstrate increased attenuation of the liver on CT - F – although Cu has high atomic number and can cause elevation of liver density on CT, this is an unusual finding perhaps because coexisting fatty infiltration diminishes hepatic parenchymal attenuation (Blackbook).
63
Q
  1. The following statements regarding cholangiography are true:
  2. The accuracy for detection of choledocholithiasis at MRCP is about 60%
  3. The main component of the MRCP pulse sequence is a short TE time
  4. Obstructed bile ducts in an atrophic liver segment will not show high signal on MRCP
  5. At serum bilirubin levels of 3-4 times above the normal range, CT cholangiography results in diagnostic images in about 80% of patients
  6. Sensitivity of helical CT intravenous cholangiography for bile duct stones detection at serum bilirubin levels above 10-15umol/L is about 95%
A

5.Sensitivity of helical CT intravenous cholangiography for bile duct stones detection at serum bilirubin levels above 10-15umol/L is about 95% - T - biliary excretion of the contrast agent is impaired in pts with serum bilirubin levels above 34-51umol/L (2-3mg/dL).

Eur Radiology 2005 findings: “CT-IVC sensitivity 95.6% and spec 94.3% in detection of choledocholithiasis in non-jaundiced patients. One of the major limitations with CT-IVC (and oral CT cholangiography) is that it often fails to opacify the bile ducts if the bilirubin is more than 2-3x normal”

  1. The following statements regarding cholangiography are true: (TW)
  2. The accuracy for detection of choledocholithiasis at MRCP is about 60% - F - Sens 81-100%, spec 85-100%; PPV 82-100%; NPV 94-100%. In view of the high NPV of MRCP, the primary utility in the setting of choledocholithiasis may not lie in the detection of CBD stones but in their exclusion. (see next qu)
  3. The main component of the MRCP pulse sequence is a short TE time - F - Imaging the pancreatic duct is depending on heavily T2WI that selectively displays static or slow-moving fluid-filled structures. T2W = long TR/TE (long TR to reduce T1 signal / longitudinal vector or spin-lattice / long TE to permit more dephasing of fat vs water / transverse vector, spin-spin), T1 = short TR/TE.
  4. Obstructed bile ducts in an atrophic liver segment will not show high signal on MRCP - F - see ans 2.
  5. At serum bilirubin levels of 3-4 times above the normal range, CT cholangiography results in diagnostic images in about 80% of patients - F - CT cholangiography performed with meglumine iotroxate (rash and nausea common side effects). Optimum images obtaioned between 5 and 30umol/L (apprx 2x normal level [2x 19umol/L]). However, high-quality images were still obtained in some patients at higher than apprx 2x normal bilirubin concentrations (up to just over 3x normal) (Aust Radiology 50;2:136-142, 2006). Major studies elsewhere looking at CT cholangiography tend to use 3x normal bilirubin as an exclusion criteria.
  6. Sensitivity of helical CT intravenous cholangiography for bile duct stones detection at serum bilirubin levels above 10-15umol/L is about 95% - T - biliary excretion of the contrast agent is impaired in pts with serum bilirubin levels above 34-51umol/L (2-3mg/dL). Eur Radiology 2005 findings: “CT-IVC sensitivity 95.6% and spec 94.3% in detection of choledocholithiasis in non-jaundiced patients. One of the major limitations with CT-IVC (and oral CT cholangiography) is that it often fails to opacify the bile ducts if the bilirubin is more than 2-3x normal”
64
Q
  1. The following statements regarding imaging of the liver are true:
  2. Segment V lies superior to segment VIII
  3. 20% of FNH are cold on sulphur colloid scan
  4. Enhancement of a nodule in the liver during arterial phase effectively excludes metastatic adenocarcinoma
  5. A haemangioma does not become progressively brighter with increasing T2 weighting in an MRI
A
  1. 20% of FNH are cold on sulphur colloid scan - T - most correct option: UTD says 80% of lesions will show active uptake of technetium sulfur colloid. Dahnert: 33% normal uptake / 33% incrased uptake (virtually diagnostic) / 33% cold spot (due to less Kupffer cells). Radiographics (1996) says 50% cold.
  2. The following statements regarding imaging of the liver are true: (TW)
  3. Segment V lies superior to segment VIII - F - anatomy
  4. 20% of FNH are cold on sulphur colloid scan - T - most correct option: UTD says 80% of lesions will show active uptake of technetium sulfur colloid. Dahnert: 33% normal uptake / 33% incrased uptake (virtually diagnostic) / 33% cold spot (due to less Kupffer cells). Radiographics (1996) says 50% cold.
  5. Enhancement of a nodule in the liver during arterial phase effectively excludes metastatic adenocarcinoma - F - adenocarcinoma = carcinoma with a glandular growth pattern microscopically. General term of cancers arising from glandular tissues. Thyroid papillary / follicular adenocarcinoma, ovarian cystadenocarcinoma, endometrial. Can be hypervascular.
  6. A haemangioma does not become progressively brighter with increasing T2 weighting in an MRI - F - “light bulb”
65
Q

42.Cholangiocarcinoma: which one is false

  1. Is extra hepatic in 90% of cases
  2. Is associated with PSC
  3. Shows early contrast enhancement
  4. Is homogeneously hypoechoic on ultrasound
A

LJS edit - statdx says 40% extrahepatic

  • LW: annoyingly, radiopedia states 80% are extra hepatic - I would still favour option 4 as least correct, as US appearances are variable depending on size and location of lesion.
  • *LJS further review. Rob considers Klatskin extrahepatic so including these 90% extrahepatic is true

4.Is homogeneously hypoechoic on ultrasound - F - mixed echoic, homo- / heterogeneous mass and dilated IHBDs

  1. Cholangiocarcinoma: (TW)
  2. Is extra hepatic in 90% of cases - T - 90% of all cholangiocarcinomas (Dahnert)
  3. Is associated with PSC - T - 10-15% of PSC’s develop cholangiocarcinomas.
  4. Shows early contrast enhancement - T - early rim-enhancement with progressive, central pathcy enhancement and IHBD dilatation. Persistent enhancing tumor on delayed phase.
  5. Is homogeneously hypoechoic on ultrasound - F - mixed echoic, homo- / heterogeneous mass and dilated IHBDs.
66
Q
  1. The following bone pathology is seen in chronic liver disease: which two are false
  2. Osteoporosis
  3. AVN of the femoral head
  4. Thickened calvarium
  5. Gout
A

4.Gout F

  1. The following bone pathology is seen in chronic liver disease: I think this question is stooopid.
  2. Osteoporosis T
  3. AVN of the femoral head T
  4. Thickened calvarium F
  5. Gout F
67
Q
  1. Hypervascular lesions in the liver include (which is false)
  2. Adenomatous hyperplastic nodules (aka dysplastic nodule)
  3. Regenerative nodules
  4. Small HCC
  5. FNH
  6. Colorectal metastasis
A

2.Regenerative nodules - F - not hypervascular, and low on T2 which is used to Dx from HCC. Can get hypovascular HCC’s developing in a dysplastic nodule that appearing as a bright T2 focus centrally in a low signal large nodule “nodule-in-a-nodule”.

  1. Hypervascular lesions in the liver include (TW)
  2. Adenomatous hyperplastic nodules (aka dysplastic nodule) - T ( if can arterial enhance) - may be bright T1.
  3. Regenerative nodules - F - not hypervascular, and low on T2 which is used to Dx from HCC. Can get hypovascular HCC’s developing in a dysplastic nodule that appearing as a bright T2 focus centrally in a low signal large nodule “nodule-in-a-nodule”.
  4. Small HCC - T - most small HCCs show intense enhancement
  5. FNH - T - central scar contains thick-walled vessels with sources from hepatic artery, which provide excellent arterial blood supply to the lesion. Slightly hypointense T1, iso T2, central scar high T2. Intense homogeneous enhancement during arterial phase.
  6. Colorectal metastasis - T
68
Q
  1. Hepatic Adenomas
  2. 30% show increased signal on T1 weighted images
  3. 50% have fat on CT
  4. 10% have calcium on CT
  5. In fibrolamellar HCC the scar enhances
  6. Doppler ultrasound shows a continuous venous trace
A
  1. 10% have calcium on CT - T – rarely calcified 10%
  2. Doppler ultrasound shows a continuous venous trace - T - colour Doppler may demonstrate a typically flat continuous or less commonly triphasic waveform
  3. Hepatic Adenomas (TW)
  4. 30% show increased signal on T1 weighted images - ?T - often hyperintense areas on T1WI (due to presence of fat-laden hepatocytes / haemorrhage) in 35-77%. (Dahnert)
  5. 50% have fat on CT - F – CT evidence of fat is seen in only 10% (RG). Dahnert says: mass of decreased density due to fat + areas of necrosis (30-40%).
  6. 10% have calcium on CT - T – rarely calcified 10%
  7. In fibrolamellar HCC the scar enhances - F – scar doesn’t enhance. Unlike FNH where delayed scans get scar enhancement (2/3rd of large FNH, and 1/3rd of smaller FNH).
  8. Doppler ultrasound shows a continuous venous trace - T - colour Doppler may demonstrate a typically flat continuous or less commonly triphasic waveform
69
Q
  1. In acute pancreatitis: which is false
  2. More than 50% of acute fluid collections result spontaneously
  3. Areas of non enhancement of the pancreas are an important prognostic factor
  4. Gall stone pancreatitis not resolving after 48 to 72 hours requires ERCP
  5. A fatty liver implies alcohol as the aetiology .
A

4.A fatty liver implies alcohol as the aetiology - F - multiple potential causes of fatty liver

  1. In acute pancreatitis: (TW)
  2. More than 50% of acute fluid collections result spontaneously - T - spontaneous regression in 40-50% (Dahnert)
  3. Areas of non enhancement of the pancreas are an important prognostic factor -T - unenhanced areas (<50HU) post IV contrast which are >3cm in size indicate areas of pancreatic necrosis.
  4. Gall stone pancreatitis not resolving after 48 to 72 hours requires ERCP - ?T - depends on a number of factors. UTD recommendations for gallstone pancreatitis - early ERCP and sphincterotomy only for those who have persistently abnormal or deteriorating liver tests of biliary sepsis. Cholecystectomy should be performed prior to discharge in patients with mild pancreatitis. ACG guidelines: ERCP for those with severe pancreatitis or cholangitis, or poor candidates for cholecystectomy, and those with strong evidence of persistent biliary obstruction. Routine ERCP avoided in pts with low to intermediate suspicion of retained bile duct stones in whom cholecystectomy is planned. AGA guidelines: pts with gallstone pancreatitis get urgent ERCP (<24h) if there is concomitant cholangitis, while early ERCP (<72h) in those with high suspicion of persistent bile duct stones.
  5. A fatty liver implies alcohol as the aetiology - F - multiple potential causes of fatty liver
70
Q
  1. Regarding cholecystostomy:
  2. Contraindicated in ascites
  3. Always cannulate the cystic duct
  4. Transhepatic approach is mandatory
  5. Indicated in acalculous cholecystitis
  6. Causes bradycardia
A
  1. Indicated in acalculous cholecystitis - T - the definitive therapy is cholecystectomy with drainage of any associated abscess. In some unstable, desperately ill patients, and Sx may be contraindicated, drainage of GB may be performed under radiologic guidance via perc cholecystostomy. In many cases percutaneous cholecystostomy is more than a temporizing measure. Subseuqent cholecystectomy is often unnecessary if the patient recovers and the underlying porblem causing the cholecystitis is no longer present.
  2. Causes bradycardia - T - major complication is vagal hypotension (bradycardia / hypotension) - occurs most commonly in cases of acute cholecystitis with markedly distended gallbladder.
  3. Regarding cholecystostomy: (TW)
  4. Contraindicated in ascites - F - if so, then the majority of ICU patients wouldn’t receive any intervention.
  5. Always cannulate the cystic duct - F - catherterisation of the common duct via the cystic duct usually is time consuming and impractical.
  6. Transhepatic approach is mandatory - F - the tract chosen depends on the anatomy and whether stone extraction is planned. Transhepatic route is associated with less risk of bile leakage, whereas a subhepatic or transperitoneal route is preferred for stone extraction through a larger tract.
  7. Indicated in acalculous cholecystitis - T - the definitive therapy is cholecystectomy with drainage of any associated abscess. In some unstable, desperately ill patients, and Sx may be contraindicated, drainage of GB may be performed under radiologic guidance via perc cholecystostomy. In many cases percutaneous cholecystostomy is more than a temporizing measure. Subseuqent cholecystectomy is often unnecessary if the patient recovers and the underlying porblem causing the cholecystitis is no longer present.
  8. Causes bradycardia - T - major complication is vagal hypotension (bradycardia / hypotension) - occurs most commonly in cases of acute cholecystitis with markedly distended gallbladder.
71
Q
  1. Regarding hepatic adenomas:
  2. SPIO
  3. Always hot on colloid scan
  4. Loses signal on out of phase imaging
  5. Usually multifocal
A

3.Loses signal on out of phase imaging - T - composed of hepatocytes containing

  1. Regarding hepatic adenomas: (JS)
  2. SPIO - F - Kupffer cells are non-functioning and reduced in number therefore no uptake of SPIO
  3. Always hot on colloid scan - F - cold on sulfur colloid scan
  4. Loses signal on out of phase imaging - T - composed of hepatocytes containing abundant fat
  5. Usually multifocal - F - solitary in 70-80%
72
Q
  1. Hypervascular liver met
  2. FNH
  3. hepatoma
  4. regenerating nodule
  5. colorectal mets
A

4.colorectal mets - T - can be hypervascular

  1. Hypervascular liver met (TW)
  2. FNH - T - central scar contains thick-walled vessels with sources from hepatic artery, which provide excellent arterial blood supply to the lesion. Slightly hypointense T1, iso T2, central scar high T2. Intense homogeneous enhancement during arterial phase.
  3. hepatoma - T - small hepatocellular adenomas enhance rapidly and are hyperattenuating relative to the liver. Peripheral enhancement may be seen as reflecting the presence of the large subcapsular feeding vessels, with a centripetal pattern of enhancement. Enhancement usually does not persist in adenomas due to arteriovenous shunting. Histo: large plates or cords of cells closely resembling normal hepatocytes, with the plates separated by dilated sinusoids. These sinusoids are equivalent ot thin-walled capillaries that are perfused by arterial pressure because adenomas lack a portal venous supply and are fed solely by peripheral arterial feeding vessels.
  4. regenerating nodule - F - differentiating factor from small HCC.
  5. colorectal mets - T - can be hypervascular
73
Q
  1. Gadolinium

1. liver excretion

A

50.Gadolinium (TW)1.liver excretion Gadolinium chelates and iodinated contrast media share similar pharmacokinetic in the liver and throughout the body. After IV injection, distributes rapidly into body’s vascular and interstitial spaces and are eliminated in unaltered form by glomerular filtration. Unlike iodine, extracellulear gadolinium chelates are not excreted by the hepatobiliary system and therefore are not observed in the gallbladder in the setting of renal failure.Primovist is a water soluble ethoxybenzyl derivative of Gc-DTPA. Taken up by hepatocytes (approx 30% dose goes to hepatocytes) and is equally excreted renal and biliary in humans. As malignant tumors (mets and most HCCs) do not contain either hepatocytes or their function is hampered - get improved liver-to-lesion contrast.

74
Q
  1. Cholangiocarcinoma
  2. enhances early
  3. 90% extrahepatic
  4. capsular retraction
A

*AJL: Robbins includes klatskin (perihilar) as extrahepatic and therefore statdx ultimately concurs that 90% are extrahepatic. **LJS agree
(also new terminology in robbins (in case it somehow crops up), intraheptic cholangiocarcinoma and extrahepatic biliary adenocarcinoma)

*LW: radiopedia sates 80% extra hepatic.
Would come down to actual question, as this is incomplete recall, I would favor there would be a more false / incorrect answer than those listed.

all true

  1. Cholangiocarcinoma (TW)
  2. enhances early - T - early rim-enhancement with progressive, central pathcy enhancement and IHBD dilatation. Persistent enhancing tumor on delayed phase.
  3. 90% extrahepatic - T - 90% of all cholangiocarcinomas (Dahnert)
  4. capsular retraction - T

Radiographics 2002:
Frequently noted ancillary findings in peripheral cholangiocarcioma include capsular retraction and dilatation and thickening of the peripheral intrahepatic bule ducts (especially when associated with clonorchiasis).
Radiographics 2009:
Cholangiocarcinoma arising from a cirrhotic liver may be surrounded by a fibrotic pseudocapsule, which is an unusual finding in a cholangiocarcinoma arising from a noncirrhotic liver. In such cases, capsular retraction is noted along the tumor surface. This capsular retraction may be seen in some HCC’s with cirrhotic stroma but is more suggestive of cholangiocarcinoma.

75
Q
  1. Hepatic adenoma
  2. 50% contains fat on CT
  3. 30% high on T1
  4. show delayed enhancement on HIDA
  5. early enhancement
A

4.early enhancement - T - small hepatocellular adenomas enhance rapidly and are hyperattenuating relative to the liver. Peripheral enhancement may be seen as reflecting the presence of the large subcapsular feeding vessels, with a centripetal pattern of enhancement. Enhancement usually does not persist in adenomas due to arteriovenous shunting. Histo: large plates or cords of cells closely resembling normal hepatocytes, with the plates separated by dilated sinusoids. These sinusoids are equivalent ot thin-walled capillaries that are perfused by arterial pressure because adenomas lack a portal venous supply and are fed solely by peripheral arterial feeding vessels.

  1. Hepatic adenoma (TW)
  2. 50% contains fat on CT - F – CT evidence of fat is seen in only 10% (RG). Dahnert says: mass of decreased density due to fat + areas of necrosis (30-40%).
  3. 30% high on T1 - ?T - often hyperintense areas on T1WI (due to presence of fat-laden hepatocytes / haemorrhage) in 35-77%. (Dahnert)
  4. show delayed enhancement on HIDA - F - Compared with normal liver, adenomas usually show absent or decreased uptake of Tc-99m sulfur colloid, reflecting the decreased number or function of Kupffer cells. Whereas hepatobiliary scintigraphy (performed with Tc-99m HIDA) is useful in Dx FNH, adenomas (do show increased activity on HIDA) however do not usually demonstrated the delayed clearance of radioactivity (“hot spots); however, neither do other hepatic neoplasms.
  5. early enhancement - T - small hepatocellular adenomas enhance rapidly and are hyperattenuating relative to the liver. Peripheral enhancement may be seen as reflecting the presence of the large subcapsular feeding vessels, with a centripetal pattern of enhancement. Enhancement usually does not persist in adenomas due to arteriovenous shunting. Histo: large plates or cords of cells closely resembling normal hepatocytes, with the plates separated by dilated sinusoids. These sinusoids are equivalent ot thin-walled capillaries that are perfused by arterial pressure because adenomas lack a portal venous supply and are fed solely by peripheral arterial feeding vessels.
76
Q
53.Haemangioma of the liver
.1.Are more common in females.
2.30% are multiple.
3.Fill in by three minutes on C.T.
4.Show decreased echogenicity when compressed at intraoperative ultrasound.
5.Are bright on T-1 images.
A

1.Are more common in females - T - F are 5x > M

  1. Haemangioma of the liver. (TW)
  2. Are more common in females - T - F are 5x > M
  3. 30% are multiple - F - multiple in up to 50% (note Dahnert says 10-20%)
  4. Fill in by three minutes on C.T. - F - delayed phase scans 5-10min post IV (note that Dahnert says complete fill-in on delayed images 3-30min post IV bolus (55-89%).
  5. Show decreased echogenicity when compressed at intraoperative ultrasound - F
  6. Are bright on T-1 images - F - isointense to blood or hypointense.
77
Q
  1. Biliary imaging.
  2. Dilatation of the gall bladder and CBD suggests a non-calculus cause.
  3. Gall bladder wall thickness is normally less than 6 mm
  4. Adenomyosis is associated with septae and polyps.
  5. The CBD is anterior to the portal vein in the hepato-duodenal ligament.
A

4.The CBD is anterior to the portal vein in the hepato-duodenal ligament - T - anterior and right, hepatic artery anterior and left.

  1. Biliary imaging (TW)
  2. Dilatation of the gall bladder and CBD suggests a non-calculus cause - T - i’m assuming this question is getting at Courvoisier’s law - ie palpable gallbladder (ie distended), painless jaundice is unlikely to be caused by gallstone. As gallstones are formed over an extended period of time, this results in a chronic, shrunken, scarred, fibrotic gallbladder which does not distend easily. Raises suspicion of pancreatic malignancy. However, i’m sure is not without exception.
    * AJL - I would favour this to be false. Acute calculous cholecystitis often has a dilated CBD as well from a distal CBD stone. (though I agree with the above point, just a poorly worded question)
  3. Gall bladder wall thickness is normally less than 6 mm - F - <3mm
  4. Adenomyosis is associated with septae and polyps - F - (adenomyosis is in the uterus). Adenomyomatosis has intramural diverticula (Rokitansky-Aschoff sinus). Comet-tail sound reverberation artifact between cholesterol crystals in RA sinuses on US.
  5. The CBD is anterior to the portal vein in the hepato-duodenal ligament - T - anterior and right, hepatic artery anterior and left.
78
Q
55.Microcystic adenoma (serous cystadenoma) of the pancreas
.1.Bright on ultrasound.
2.Best treated by surgery.
3.Young females.
4.Benign tumour.
5.May show foci of calcification.
A
  1. Benign tumour - T - benign tumor with no malignant potential (Blackbook)
  2. May show foci of calcification - T - calcificaiton within a central scar may be seen
  3. Microcystic adenoma (serous cystadenoma) of the pancreas. (TW)
  4. Bright on ultrasound - F - hypo echoic
  5. Best treated by surgery - F - if definite Dx is made, then management is determined by symptoms, progression, and lesion location. If symptomatic or enlarging then resect. Small asymptomatic, and nonenlarging lesions can be observed since the risk of malignant change is small (UTD).
  6. Young females. - F - Middle and elderly age group more common (avg 65yo). Females 4x > Males. Not to be confused with solid and papillary neoplasm (<35yo, F 9x > M, African-American or other non-caucasians groups).
  7. Benign tumour - T - benign tumor with no malignant potential (Blackbook)
  8. May show foci of calcification - T - calcificaiton within a central scar may be seen
79
Q
  1. With colour doppler
  2. Artefactual absence of flow occurs when a vessel lies at 90 degrees to the incident beam
  3. The common bile duct is blue by convention
  4. The flow image is usually based on peak doppler shift and is therefore quantitative
  5. The shades of colour are a very accurate representation of velocity differences
  6. Slight transducer movement in venous studies produces “colour out”
A
  1. Artefactual absence of flow occurs when a vessel lies at 90 degrees to the incident beam - T - Doppler shift frequence (Fd) is the transmitted frequency (Fo) minus the received frequency (Fr). Fd = Fo - Fr. This equals 2 Fo μ(cosθ)/v (where μ = reflector velocity and v = sound velocity). cosθ = 0 when θ = 90 degrees.
  2. With colour doppler (TW)
  3. Artefactual absence of flow occurs when a vessel lies at 90 degrees to the incident beam - T - Doppler shift frequence (Fd) is the transmitted frequency (Fo) minus the received frequency (Fr). Fd = Fo - Fr. This equals 2 Fo μ(cosθ)/v (where μ = reflector velocity and v = sound velocity). cosθ = 0 when θ = 90 degrees.
  4. The common bile duct is blue by convention - F - strange option.
  5. The flow image is usually based on peak doppler shift and is therefore quantitative - F - the colour shade for each pixel is based on the mean frequency shift arising from that pixel.
  6. The shades of colour are a very accurate representation of velocity differences - F - colour doppler provides a real-time image. In practice, it is often used to identify vessels, or to identify focal areas of flow distrubances, and then waveforms from these areas are obtained with pulsed Doppler analysis.
  7. Slight transducer movement in venous studies produces “colour out” - F - couldn’t find anything on this
80
Q
  1. Portal vein thrombosis may be associated with (which is false)
  2. Hepatoma
  3. Schistosomiasis
  4. Actinomycosis
  5. Eclampsia
  6. Pancreatitis
A

*AJL - Eclampsia has thrombocytopaenia (HELLP syndrome) and therefore portal vein thrombosis is less likely, likely overcomes the prothrombotic effects of being pregnant.

2.Schistosomiasis - F - can cause presinusoidal portal obstruction by blocking intrahepatic portal venules with parasite eggs. It does not cause extrahepatic protal vein obstruction, though the clinical manifestations are similar (eMed).
*LW:
Agree with schisto statement, with below reasoning:
–> Among adults with chronic infection, the predominant pathologic process consists of collagen deposition in the periportal spaces by activated hepatic stellate cells, which causes periportal fibrosis (also known as Symmers’ pipestem fibrosis).
–> This leads to occlusion of the portal veins, portal hypertension with splenomegaly, portocaval shunting, and gastrointestinal varices.
–> no mention of actual thrombosis, or of the extra hepatic portal vein.
—> therefore, this is my preferred option of FALSE.

81
Q
  1. Gallium 67 scanning . (which is false)
  2. Nasopharyngeal uptake occurs commonly
  3. Spienic uptake is usually greater than liver uptake
  4. Gallium is usually taken up by necrotic tumours
  5. Gallium rapidly binds to transferrin
  6. Increased bone uptake is usually seen following cytotoxic therapy for soft tissue tumours
A
  1. Spienic uptake is usually greater than liver uptake - F
  2. Increased bone uptake is usually seen following cytotoxic therapy for soft tissue tumours - F - previous chemotheraphy and radiation therapy may decrease Ga uptake.
  3. Gallium 67 scanning . (TW)
  4. Nasopharyngeal uptake occurs commonly - T
  5. Spienic uptake is usually greater than liver uptake - F
  6. Gallium is usually taken up by necrotic tumours - T - rim uptake DDx abscess

.4.Gallium rapidly binds to transferrin - T - analogue of ferric ion

5.Increased bone uptake is usually seen following cytotoxic therapy for soft tissue tumours - F - previous chemotheraphy and radiation therapy may decrease Ga uptake.
Gallium-67 Citrate - acts as an analogue of ferric ion. Ga-67 is bound to iron-binding sites of various proteins (strongest bond with plasma transferrin, lactoferrin in tissue). Binding sites LFT’S = lactoferrin, ferritin, transferrin, siderophores (bacteria). Uptake at 24h - most intense in reticuloendothelia system: liver, spleen (4%), bone marrow, bowel wall, renal cortx, nasal mucosa, lacrima + salivary glands, blood pool (20%), lung, breasts. Uptake at 72h - 75% of dose remains in body its activity distributed amont soft tissue (orbit, nasal mucosa, large bowel), liver, bone, or bone marrow; kidney activity no longer detectable; lacrimal + salivary glands may still be prominent.

82
Q
  1. Cholelithiasis is associated with (which is false)
  2. Haemolytic anaemia
  3. Cystic fibrosis
  4. Wilson’s disease
  5. Gout
  6. Ellis van Creveld syndrome
A
  1. Gout - F - don’t think they are directly linked, however, hemolysis can cause both.
  2. Ellis van Creveld syndrome - F - chondroectodermal dysplasia, AR skeletal dysplasia.
  3. Cholelithiasis is associated with (TW)
  4. Haemolytic anaemia - T - sickle cell disease, hereditary spherocytosis, thalassemia, pernicious anemia, prosthetic cardiac valves, mitral stenosis, cirrhosis (hemolysis secondary to hypersplenism), Rhesus / AGO blood group incompatability.
  5. Cystic fibrosis - T - gallstones in older children assoc with CF (Dahnert).
  6. Wilson’s disease - T - cholelithiasis is relatively common in adolescents with Wilson’s disease and results from ongoing hemolysis in the presence of cirrhosis (Liver Disease in Children F.Suchy).
  7. Gout - F - don’t think they are directly linked, however, hemolysis can cause both.
  8. Ellis van Creveld syndrome - F - chondroectodermal dysplasia, AR skeletal dysplasia.